[obm-l] Re: [obm-l] Re: [obm-l] Combinatória

2024-03-13 Por tôpico Pedro Júnior
Eu pensei sim, mas e os casos do tipo ACCACAC. Esse caso não entra na conta
6! - 2* 3!* 3!.

Em qua., 13 de mar. de 2024 às 09:09, Claudio Buffara <
claudio.buff...@gmail.com> escreveu:

> Pense no oposto: de quantas maneiras as crianças e adultos podem se sentar
> separados uns dos outros.
>
> On Wed, Mar 13, 2024 at 8:39 AM Pedro Júnior 
> wrote:
>
>> Olá pessoal, bom dia.
>> Alguém poderia me ajudar nesse problema?
>>
>> Seis poltronas enfileiradas em um cinema e entram 3 adultos e 3 crianças.
>> De quantas maneiras podem sentar-se 2 crianças juntas e dois adultos juntos?
>>
>>
>> Desde já fico grato!
>>
>> --
>> Esta mensagem foi verificada pelo sistema de antivírus e
>> acredita-se estar livre de perigo.
>
>
> --
> Esta mensagem foi verificada pelo sistema de antivírus e
> acredita-se estar livre de perigo.



-- 

Pedro Jerônimo S. de O. Júnior

Professor de Matemática

Geo João Pessoa – PB

-- 
Esta mensagem foi verificada pelo sistema de antiv�rus e
 acredita-se estar livre de perigo.



[obm-l] Combinatória

2024-03-13 Por tôpico Pedro Júnior
Olá pessoal, bom dia.
Alguém poderia me ajudar nesse problema?

Seis poltronas enfileiradas em um cinema e entram 3 adultos e 3 crianças.
De quantas maneiras podem sentar-se 2 crianças juntas e dois adultos juntos?


Desde já fico grato!

-- 
Esta mensagem foi verificada pelo sistema de antiv�rus e
 acredita-se estar livre de perigo.



Re: [obm-l] Divisibilidade, pedido de esclarecimento

2024-03-04 Por tôpico Pedro José
Bom dia!
Mas provar que ocorrendo as duas está certo, não é o que foi pedido.
Pode ser que ocorrendo as duas esteja OK e também que haja pelo menos um
caso, que dá certo para a primeira assertiva e não ocorre para a segunda ou
pode ter pelo menos um caso que ocorra para a segunda e não ocorra para
primeira, já é suficiente para furar.
O certo é:
supor (i) e mostrar que ocorre(ii) e depois provar a volta, supor (ii) e
mostrar que ocorre (i).
(i) 9r + 5s | 17. 17s + 17r | 17 (iii) logo 4*(i)-2(iii) ==>  2r - 14s | 17
(iv).
Como 17s! 17 (v); (1v)+ (v) ==> 2r+3s | 17. Provada a ida. 17
2r +3s |17 (ii) . Mas 17r + 17 s | 17 (iii). (iii)- 4*(i) ==> 9r +5s | 17
Provada a volta.
logo 9r + 5s | 17 <=> 2r+ 3s | 17 C.Q.D.


Cordialmente,
PJMS

Em sáb., 2 de mar. de 2024 às 12:37, Marcone Borges <
marconeborge...@hotmail.com> escreveu:

> Sendo r e s inteiros, mostre que 9r +5s divide 17 se, e somente se, 2r +
> 3s divide 17.
> De 9r + 5s ==0(mod 17), assim como de 2r + 3s ==0(mod17), segue que
> r==7s (mod17). Daí sai a resposta.
> Ou podemos mostrar o que foi pedido usando 9r + 5r +4(2r +3s) = 17(r + s)
> Mas, do ponto de vista de quem elaborou a questão, por que vincular essas
> expressões ao fato de que quando uma for um múltiplo de 17 a outra também
> será?
>
> --
> Esta mensagem foi verificada pelo sistema de antivírus e
> acredita-se estar livre de perigo.
>

-- 
Esta mensagem foi verificada pelo sistema de antiv�rus e
 acredita-se estar livre de perigo.



[obm-l] Re: [obm-l] Re: [obm-l] Desigualdade e frações

2024-02-27 Por tôpico Pedro Júnior
Desculpas, Cláudio. É isso mesmo, com "a" e "b" inteiros e positivos.

Obrigado pela brilhante solução.

Em ter, 27 de fev de 2024 01:41, Claudio Buffara 
escreveu:

> Deveria ser a e b inteiros positivos, não?
> Pois se forem inteiros sem restrição, então como 2022/2023 < 2022,5/2023,5
> < 2023/2024, bastaria tomar a sequência:
> a(n) = -20225*n  e  b(n) = -20235*n.
> Daí teríamos 2022/2023 < a(n)/b(n) < 2023/2024 e a sequência a(n)+b(n)
> seria ilimitada inferiormente.
>
> Assim, suponhamos que a e b sejam inteiros positivos.
> 2022/2023 < a/b < 2023/2024 implica que b > a+1, já que a sequência
> (n/(n+1)) é crescente.
> Além disso, usando razões e proporções, achamos que:
> 2022 < a/(b-a) < 2023 < b/(b-a) < 2024
> ==> para que a+b seja o menor possível, b-a deverá ser o menor possível.
> E o menor valor possível de b-a é 2.
> Usando frações equivalentes, dá pra escrever 4044/4046 < a/b < 4046/4048 e
> daí teríamos uma única fração a/b com b - a = 2.
> Seria a/b = 4045/4047 ==> a+b mínimo = 8092.
>
> []s,
> Claudio.
>
>
>
>
> On Mon, Feb 26, 2024 at 10:12 PM Pedro Júnior 
> wrote:
>
>> Quem puder me ajudar, fixo grato.
>>
>> Sejam a e b dois números inteiros. Sabendo que 2022/2023 < a/b <
>> 2023/2024, determine o menos calor da soma a + b.
>>
>> --
>> Esta mensagem foi verificada pelo sistema de antivírus e
>> acredita-se estar livre de perigo.
>
>
> --
> Esta mensagem foi verificada pelo sistema de antivírus e
> acredita-se estar livre de perigo.

-- 
Esta mensagem foi verificada pelo sistema de antiv�rus e
 acredita-se estar livre de perigo.



[obm-l] Desigualdade e frações

2024-02-26 Por tôpico Pedro Júnior
Quem puder me ajudar, fixo grato.

Sejam a e b dois números inteiros. Sabendo que 2022/2023 < a/b < 2023/2024,
determine o menos calor da soma a + b.

-- 
Esta mensagem foi verificada pelo sistema de antiv�rus e
 acredita-se estar livre de perigo.



Re: [obm-l] OBM 88 Problema 6.

2023-12-30 Por tôpico Pedro José
Boa noite!
Cláudio, minha preocupação é com a solução em si da equação.
O problema original pede que demonstre que k é um quadrado perfeito. Todas
soluções que vi são baseadas nas relações de Girad ou Vieta's fórmula como
chamam lá fora.
Eu parti do conhecimento de que k tem de ser quadrado perfeito.
Consegui provar que tirando as soluções triviais a=0 ou b=0 ou a=b=1
b>=raiz(k)
Aí achei a primeira solução para a equação, sem perda de generalidade,
considerei a>b, a=b só ocorre para a=b=1 ou a=b=0. Lá fora acho que nem
consideram 0 natural. Seguem a risca como foi o postulado de Peano.
Então para cada k=w^2 com w>1
Tem um conjunto com uma sequência infinita de soluções.
Sk={si=(ai,bi,k): i natural e i>=1| s1=(w^3,w,w^2) e si+1=(ai*w^2-bi, ai,
w^2).
Consigo provar que todos termos da sequência são soluções.
Não consigo provar que se há uma solução (a*,b*, k*) então (a*,b*,k*) ou
(b*,a*, k*) pertence a sequência Sk para k=w^2.
Eu não acho a solução da equação, só do problema como foi pedido, mostrar
que k é um QP, sem no entanto achar todas as soluções

Cordialmente,
PJMS

Em sex., 29 de dez. de 2023 09:18, Claudio Buffara <
claudio.buff...@gmail.com> escreveu:

> Dá um Google em "IMO 88".
> Vai ter até vídeo com a solução deste problema.
>
> On Thu, Dec 28, 2023 at 4:35 PM Pedro José  wrote:
>
>> Boa tarde!
>> Com referência a esse problema criei uma conjectura, não consegui provar
>> com a pretensão de abranger todas as soluções da equação:
>>
>> (a^2+b^2)/(ab+1)= k, com a,b,k Naturais e a>1, b>1 e k>1 Fiz essa
>> restrição para retirar as soluções triviais.
>> E SPG considerei a>b, já que a=b só ocorre para a=b=1, que está fora pela
>> restrição acima e por ser uma equação simétrica em relação à a e b.
>> O problema era provar que k era um quadrado perfeito.
>> Gostaria de saber se alguém teria conhecimento da resolução em si do
>> problema, i.e., quais ternos (a*,b*,k*) são solução da equação.
>> Caso ninguém tenha resolvido a equação, ainda, gostaria como faço para
>> dar divulgação da minha conjectura, onde tenho a pretenção de ter
>> encontrado todas as soluções possíveis para a equação em epígrafe, no
>> Universo dos Naturais, com a restrição a>1, b>1 e K>1.
>>
>> Agradeço quem puder me orientar.
>>
>> Cordialmente,
>> PJMS
>>
>> --
>> Esta mensagem foi verificada pelo sistema de antivírus e
>> acredita-se estar livre de perigo.
>
>
> --
> Esta mensagem foi verificada pelo sistema de antivírus e
> acredita-se estar livre de perigo.

-- 
Esta mensagem foi verificada pelo sistema de antiv�rus e
 acredita-se estar livre de perigo.



Re: [obm-l] OBM 88 Problema 6.

2023-12-28 Por tôpico Pedro José
Peço máxima vênia. Nem.reparata que fizera uma referência errada. OBM ao
invés de IMO. Interpretei erroneamente como uma censura. Só depois é que
reparei que falhará na referência.
Minhas escusas.

Cordialmente, PJMS.

Em qui., 28 de dez. de 2023 19:47, Anderson Torres <
torres.anderson...@gmail.com> escreveu:

>
>
> Em qui, 28 de dez de 2023 19:01, Pedro José 
> escreveu:
>
>> E daí?
>>
>
> E daí e daí?
>
>
>> Em qui., 28 de dez. de 2023 18:42, Anderson Torres <
>> torres.anderson...@gmail.com> escreveu:
>>
>>> Isso não é da OBM mas da IMO
>>>
>>> Em qui, 28 de dez de 2023 16:35, Pedro José 
>>> escreveu:
>>>
>>>> Boa tarde!
>>>> Com referência a esse problema criei uma conjectura, não consegui
>>>> provar com a pretensão de abranger todas as soluções da equação:
>>>>
>>>> (a^2+b^2)/(ab+1)= k, com a,b,k Naturais e a>1, b>1 e k>1 Fiz essa
>>>> restrição para retirar as soluções triviais.
>>>> E SPG considerei a>b, já que a=b só ocorre para a=b=1, que está fora
>>>> pela restrição acima e por ser uma equação simétrica em relação à a e b.
>>>> O problema era provar que k era um quadrado perfeito.
>>>> Gostaria de saber se alguém teria conhecimento da resolução em si do
>>>> problema, i.e., quais ternos (a*,b*,k*) são solução da equação.
>>>>
>>>
>>> Sim, o próprio método de resolução por descenso provê um método de
>>> listagem das soluções.
>>>
>>> Caso ninguém tenha resolvido a equação, ainda, gostaria como faço para
>>>> dar divulgação da minha conjectura, onde tenho a pretenção de ter
>>>> encontrado todas as soluções possíveis para a equação em epígrafe, no
>>>> Universo dos Naturais, com a restrição a>1, b>1 e K>1.
>>>>
>>>> Agradeço quem puder me orientar.
>>>>
>>>> Cordialmente,
>>>> PJMS
>>>>
>>>> --
>>>> Esta mensagem foi verificada pelo sistema de antivírus e
>>>> acredita-se estar livre de perigo.
>>>
>>>
>>> --
>>> Esta mensagem foi verificada pelo sistema de antivírus e
>>> acredita-se estar livre de perigo.
>>
>>
>> --
>> Esta mensagem foi verificada pelo sistema de antivírus e
>> acredita-se estar livre de perigo.
>
>
> --
> Esta mensagem foi verificada pelo sistema de antivírus e
> acredita-se estar livre de perigo.

-- 
Esta mensagem foi verificada pelo sistema de antiv�rus e
 acredita-se estar livre de perigo.



Re: [obm-l] OBM 88 Problema 6.

2023-12-28 Por tôpico Pedro José
E daí?

Em qui., 28 de dez. de 2023 18:42, Anderson Torres <
torres.anderson...@gmail.com> escreveu:

> Isso não é da OBM mas da IMO
>
> Em qui, 28 de dez de 2023 16:35, Pedro José 
> escreveu:
>
>> Boa tarde!
>> Com referência a esse problema criei uma conjectura, não consegui provar
>> com a pretensão de abranger todas as soluções da equação:
>>
>> (a^2+b^2)/(ab+1)= k, com a,b,k Naturais e a>1, b>1 e k>1 Fiz essa
>> restrição para retirar as soluções triviais.
>> E SPG considerei a>b, já que a=b só ocorre para a=b=1, que está fora pela
>> restrição acima e por ser uma equação simétrica em relação à a e b.
>> O problema era provar que k era um quadrado perfeito.
>> Gostaria de saber se alguém teria conhecimento da resolução em si do
>> problema, i.e., quais ternos (a*,b*,k*) são solução da equação.
>>
>
> Sim, o próprio método de resolução por descenso provê um método de
> listagem das soluções.
>
> Caso ninguém tenha resolvido a equação, ainda, gostaria como faço para dar
>> divulgação da minha conjectura, onde tenho a pretenção de ter encontrado
>> todas as soluções possíveis para a equação em epígrafe, no Universo dos
>> Naturais, com a restrição a>1, b>1 e K>1.
>>
>> Agradeço quem puder me orientar.
>>
>> Cordialmente,
>> PJMS
>>
>> --
>> Esta mensagem foi verificada pelo sistema de antivírus e
>> acredita-se estar livre de perigo.
>
>
> --
> Esta mensagem foi verificada pelo sistema de antivírus e
> acredita-se estar livre de perigo.

-- 
Esta mensagem foi verificada pelo sistema de antiv�rus e
 acredita-se estar livre de perigo.



[obm-l] OBM 88 Problema 6.

2023-12-28 Por tôpico Pedro José
Boa tarde!
Com referência a esse problema criei uma conjectura, não consegui provar
com a pretensão de abranger todas as soluções da equação:

(a^2+b^2)/(ab+1)= k, com a,b,k Naturais e a>1, b>1 e k>1 Fiz essa restrição
para retirar as soluções triviais.
E SPG considerei a>b, já que a=b só ocorre para a=b=1, que está fora pela
restrição acima e por ser uma equação simétrica em relação à a e b.
O problema era provar que k era um quadrado perfeito.
Gostaria de saber se alguém teria conhecimento da resolução em si do
problema, i.e., quais ternos (a*,b*,k*) são solução da equação.
Caso ninguém tenha resolvido a equação, ainda, gostaria como faço para dar
divulgação da minha conjectura, onde tenho a pretenção de ter encontrado
todas as soluções possíveis para a equação em epígrafe, no Universo dos
Naturais, com a restrição a>1, b>1 e K>1.

Agradeço quem puder me orientar.

Cordialmente,
PJMS

-- 
Esta mensagem foi verificada pelo sistema de antiv�rus e
 acredita-se estar livre de perigo.



Re: [obm-l] Conjuntos

2023-09-21 Por tôpico Pedro José
Boa tarde!
Vou considerar 3 números mesmo.
3, 3, 3 é um número só repetido três  vezes.
Os três números obrigatoriamente estarão em P.A. Então usando a menor razão
r <>0;
temos r=1
{1,2,3} {2,3,4}...{2020, 2021, 2022}
{2021, 2022, 2023} temos 2021 conjuntos para r=1.
É fácil observar que para r=2 o último
conjunto será  {2019, 2021, 2023} assim sendo teremos 2019 conjuntos.
E a cada unidade que aumentamos em r diminuímos em 2 o número de conjuntos
Até que chegaremos a um conjunto apenas. {1, 1012, 2023}
Logo o número de conjuntos N será a soma de:
N= 1 + 3 +5+..2019+2021, que é uma PA de razão 2.
seja n o número de termos da PA
n=(2021-1)/2+1=1011
N=(1+2021)*1011/2=1.022.121

Cordialmente,
PJMS


Em ter., 8 de ago. de 2023 19:53, Anderson Torres <
torres.anderson...@gmail.com> escreveu:

> mande uma vez somente.
>
> Em ter, 8 de ago de 2023 12:33, Jamil Silva 
> escreveu:
>
>> Quantos conjuntos de três números inteiros positivos menores ou iguais a
>> 2023 contêm a média aritmética de seus elementos ?
>>
>> --
>> Esta mensagem foi verificada pelo sistema de antivírus e
>> acredita-se estar livre de perigo.
>>
>
> --
> Esta mensagem foi verificada pelo sistema de antivírus e
> acredita-se estar livre de perigo.

-- 
Esta mensagem foi verificada pelo sistema de antiv�rus e
 acredita-se estar livre de perigo.



[obm-l] Cone Sul

2023-03-13 Por tôpico Pedro Júnior
Olá pessoal, muito bom dia.
Gostaria de saber se tem um site oficial da competição "Cone Sul de
Matemática"? Procurei o banco de provas pelo Google e não encontrei. Me
remete ao site da OBM e também não vi por lá.

Desde já fico grato.

-- 
Esta mensagem foi verificada pelo sistema de antiv�rus e
 acredita-se estar livre de perigo.



[obm-l] Re: [obm-l] Seria por distribuição binomial ou alguma recorrência

2023-02-28 Por tôpico joao pedro b menezes
A probabilidade do estudante acertar um número n de questões é [ (1/5)^n *
(4/5)^(25-n) ] * n!*(25-n)!/25! . ( o primeiro segmento, separo por [
...], indica a probabilidade de ele acertar n questões em uma ordem
definida, enquanto a segunda parte se refere ao número de combinações
possíveis em que ele acerta n questões ) . Agora é somar e fatorar

On Tue, Feb 28, 2023, 11:52 Bianca Flores  wrote:

> Alguém poderia ajudar com essa questão: estou frustrada porque não consigo
> chegar ao gabarito E.
>
> Um estudante preenche, aleatoriamente e de forma independente cada uma das
> questões, um exame de múltipla escolha com 5 respostas possíveis (das quais
> apenas uma é correta) para cada uma de 25 questões. A probabilidade que ele
> acerte um número par de questões é dada por:
>
> (A)(1-(4/5)^25)/2
> (B)(1-(3/5)^25)/2
> (C)((3/5)^25)/2
> (D)(1+(4/5)^25)/2
> (E)(1+(3/5)^25)/2
>
> Tento de todas as formar usar a distribuição binomial, alguma recorrência,
> mas sem sucesso.
> Bianca
>
> --
> Esta mensagem foi verificada pelo sistema de antivírus e
> acredita-se estar livre de perigo.
>

-- 
Esta mensagem foi verificada pelo sistema de antiv�rus e
 acredita-se estar livre de perigo.



[obm-l] Re: [obm-l] Re: [obm-l] Caracterização de Inteiros

2022-11-15 Por tôpico Pedro José
Obrigado a você e ao Cláudio. Mas não sou criativo para inventar. Mas já vi
que terei que fazer uma homotetia, para as classes de equivalência para
representar só como um número e não como um par, creio eu.

Cordialmente,
PJMS

Em ter., 15 de nov. de 2022 às 16:00, Anderson Torres <
torres.anderson...@gmail.com> escreveu:

>
>
> Em ter, 15 de nov de 2022 14:33, Pedro José 
> escreveu:
>
>> Boa tarde!
>> Para os |Naturais, temos os postulados de Peano.
>>
>> Para os Inteiros há alguma formalização?
>>
>
> invente uma!
>
> Pode ser por exemplo o conjunto de pares (p,q) tais que p-q é constante.
>
> ou melhor (p1,q1)=(p2,q2) se e só se p1+q2=p2+q1.
>
>
>> Acho pobre dizer que é necessário ter outros números devido ao problema
>> de fechamento nos naturais para a subtração que é fato e daí introduzir os
>> simétricos que são inteiros e ainda não foram caracterizados.
>>
>> No meu antigo ginásio aprendi que os Reais era a união dos conjuntos
>> disjuntos irracionais e racionais. Os racionais haviam sido bem definidos.
>> Aí questionei e o que são irracionais? resposta: são os Reais que não são
>> racionais, os que não podem ser escritos na forma p/q p e q inteiros e
>> q<>0. Mas me deram um tombo. Definiram os |Reais com base nos irracionais e
>> os irracionais com base nos |Reais. 3 +2i também não pode ser inscrito na
>> forma p/q. Só mais tarde no científico, é que meu professor definiu
>> irracional como um número que não podia ser escrito na forma p/q e cuja
>> representação decimal tinha uma infinidade de algarismos, sem haver uma
>> periodicidade.
>> Na época foi o maior nó que tive com a matemática. O mestre demonstrou
>> que os racionais eram densos, mas entre eles ainda cabiam os irracionais.
>> Não satisfeito mostrou que os racionais eram enumeráveis e por absurdo
>> mostrou que os |Reais não. Não satisfeito mostrou que a cardinalidade do
>> intervalo [0,1] era maior que a dos |Naturais. Não conseguia conceber que
>> havia um infinito maior que outro. Outra coisa que demorei a aceitar,mesmo
>> vendo a bijeção, era que os inteiros e naturais tinham a mesma
>> cardinalidade. Na minha cabeça, os inteiros têm todos os naturais ainda
>> sobram os negativos, como é igual?
>> Hoje, depois de velho, arrumei uma enteada, que muito me pergunta e estou
>> enrolado. Para dar um ar de superioridade, questionei se conhecia os
>> inteiros de Gaus, que 5 não era primo nos inteiros de Gaus. Estrepei-me, a
>> danada foi pesquisar e me questiona sobre o que não tenho um domínio pleno.
>> Em suma, como apresentei a ela os postulados de Peano para a
>> caracterização dos Naturais, ela me cobra por algo semelhante para os
>> Inteiros, e não sei responder.
>> HELP! SOCORRO! AU SECOURS! AYUDA! AIUTO! HILFE!
>> Cordialmente,
>> PJMS
>>
>> --
>> Esta mensagem foi verificada pelo sistema de antivírus e
>> acredita-se estar livre de perigo.
>
>
> --
> Esta mensagem foi verificada pelo sistema de antivírus e
> acredita-se estar livre de perigo.

-- 
Esta mensagem foi verificada pelo sistema de antiv�rus e
 acredita-se estar livre de perigo.



[obm-l] Caracterização de Inteiros

2022-11-15 Por tôpico Pedro José
Boa tarde!
Para os |Naturais, temos os postulados de Peano.

Para os Inteiros há alguma formalização?

Acho pobre dizer que é necessário ter outros números devido ao problema de
fechamento nos naturais para a subtração que é fato e daí introduzir os
simétricos que são inteiros e ainda não foram caracterizados.

No meu antigo ginásio aprendi que os Reais era a união dos conjuntos
disjuntos irracionais e racionais. Os racionais haviam sido bem definidos.
Aí questionei e o que são irracionais? resposta: são os Reais que não são
racionais, os que não podem ser escritos na forma p/q p e q inteiros e
q<>0. Mas me deram um tombo. Definiram os |Reais com base nos irracionais e
os irracionais com base nos |Reais. 3 +2i também não pode ser inscrito na
forma p/q. Só mais tarde no científico, é que meu professor definiu
irracional como um número que não podia ser escrito na forma p/q e cuja
representação decimal tinha uma infinidade de algarismos, sem haver uma
periodicidade.
Na época foi o maior nó que tive com a matemática. O mestre demonstrou que
os racionais eram densos, mas entre eles ainda cabiam os irracionais. Não
satisfeito mostrou que os racionais eram enumeráveis e por absurdo mostrou
que os |Reais não. Não satisfeito mostrou que a cardinalidade do intervalo
[0,1] era maior que a dos |Naturais. Não conseguia conceber que havia um
infinito maior que outro. Outra coisa que demorei a aceitar,mesmo vendo a
bijeção, era que os inteiros e naturais tinham a mesma cardinalidade. Na
minha cabeça, os inteiros têm todos os naturais ainda sobram os negativos,
como é igual?
Hoje, depois de velho, arrumei uma enteada, que muito me pergunta e estou
enrolado. Para dar um ar de superioridade, questionei se conhecia os
inteiros de Gaus, que 5 não era primo nos inteiros de Gaus. Estrepei-me, a
danada foi pesquisar e me questiona sobre o que não tenho um domínio pleno.
Em suma, como apresentei a ela os postulados de Peano para a caracterização
dos Naturais, ela me cobra por algo semelhante para os Inteiros, e não sei
responder.
HELP! SOCORRO! AU SECOURS! AYUDA! AIUTO! HILFE!
Cordialmente,
PJMS

-- 
Esta mensagem foi verificada pelo sistema de antiv�rus e
 acredita-se estar livre de perigo.



[obm-l] Re: [obm-l] Re: [obm-l] Questão de probabilidade

2022-06-20 Por tôpico Pedro José
Eu na minha humilde opinião creio que a probabilidade exista quando pode
ser uma coisa ou outra. No caso já é definido o que os animais são. Então
já está tudo errado. A questão seria viável se dessem esses limitantes para
uma criança que pintaria os desenhos dos animais. Aí sim há probabilidade.

Em sáb., 18 de jun. de 2022 03:33, Rogerio Ponce da Silva <
abrlw...@gmail.com> escreveu:

> Ola' Vanderlei e pessoal da lista!
>
> Sem perda de generalidade, podemos imaginar que vamos fazer o seguinte:
>
> - uma pintura preta em um dos caes, escolhido aleatoriamente
>
> - uma pintura "malhada" em um dos animais, escolhido aleatoriamente entre
> os 7 animais nao pintados
>
> - duas pintura pretas, em dois animais, escolhidos aleatoriamente entre os
> 6 animais restantes,
>
> - quatro pinturas brancas nos 4 animais restantes
>
>
> Analisando a afirmacao 04, por exemplo, verificamos que, no segundo passo
> (pintura malhada) existem 4 opcoes de cachorro e 3 opcoes de gato.
>
> Assim, a probabilidade de haver um cachorro malhado (4/7) e' maior que a
> probabilidade de haver um gato malhado (3/7).
> Portanto, a afirmacao 04 esta' correta.
> (e o gabarito esta' errado).
>
> []'s
> Rogerio Ponce
>
>
>
> On Wed, Mar 16, 2022 at 8:08 AM Professor Vanderlei Nemitz <
> vanderma...@gmail.com> wrote:
>
>> Bom dia!
>> Na questão a seguir, do vestibular da UEM, penso que o espaço amostral
>> tem 105 elementos, pois um cachorro é preto (desconsideramos esse). Porém,
>> com esse pensamento, não consigo obter o gabarito, que diz que 02 e 16 são
>> corretas.
>> Alguém poderia ajudar?
>> Muito obrigado!
>>
>> *Em um pet shop há 3 gatos e 5 cães. Sabemos que 3 desses animais são
>> pretos, 4 são brancos e 1 é malhado. Além disso, pelo menos 1 cachorro é
>> preto. Assinale o que for correto. *
>> *01) A probabilidade de haver exatamente 1 cachorro preto é de 1/6. *
>> *02) A probabilidade de haver pelo menos 1 gato branco e pelo menos 2
>> cachorros brancos é de 2/3.*
>> *04) A probabilidade de haver um cachorro malhado é maior do que a
>> probabilidade de haver um gato malhado. *
>> *08) Se um animal for escolhido ao acaso, a probabilidade de ele ser um
>> cachorro preto é de 1/8. *
>> *16) Se um animal for escolhido ao acaso, a probabilidade de ele ser um
>> gato malhado é de 1/16.   *
>>
>> --
>> Esta mensagem foi verificada pelo sistema de antivírus e
>> acredita-se estar livre de perigo.
>
>
> --
> Esta mensagem foi verificada pelo sistema de antivírus e
> acredita-se estar livre de perigo.

-- 
Esta mensagem foi verificada pelo sistema de antiv�rus e
 acredita-se estar livre de perigo.



[obm-l] Re: [obm-l] Re: [obm-l] Dúvida e ajuda.

2022-04-08 Por tôpico Pedro José
Grato a todos!
Já, já tenho de voltar ao trabalho.
Depois dou uma olhada.
Mas achei a demonstração usando casa de pombos, simples e prática.
Já que tem de haver um p/q com pp temos w=x+p/q,
onde x é a parte inteira de w/q, então pq e os restos só podem q-1, uma hora tem de
repetir e aí volta a sequência.
Mas saindo do trabalho dou uma olhada.
Mais uma vez, minha gratidão.

Cordialmente,
PJMS



Em sex., 8 de abr. de 2022 às 13:02, Claudio Buffara <
claudio.buff...@gmail.com> escreveu:

> A volta é fácil também: ao calcular a representação decimal de a/b (a e b
> naturais), nas divisões sucessivas por b só existem b-1 restos possíveis
> (resto = 0 em alguma etapa implica numa decimal finita) e, portanto, após
> não mais do que b-1 divisões, um resto vai se repetir, marcando o início de
> um novo período na representação decimal.
>
> Agora, suponha que  X =
> 0,123456789112233445566778899111222333444555666777888999... seja racional.
> Então existirão n e p naturais tais que, a partir da n-ésima casa decimal
> (1/10^n), os algarismos de X vão se repetir numa sequência com período p.
>
> Mas, pela lei de formação de X, vai existir uma sequência de n+p+1
> algarismos iguais a 1, e esta sequência vai começar após a n-ésima casa
> decimal.
> Ou seja, a sequência vai estar incluída na parte periódica da
> representação decimal de X.
> Mas como o período é p, isso implica que a parte periódica teria que
> ser 111..11 (p algarismos 1) ==> contradição à lei de formação de X.
>
> []s,
> Claudio.
>
>
> On Fri, Apr 8, 2022 at 11:17 AM Pedro José  wrote:
>
>> Bom dia!
>> Posso concluir que um número representado por uma infinidade de
>> algarismos decimais é racional se e somente se tem um período de repetições
>> desses algarismos?
>> A ida é fácil se tiver o período é racional.
>> Já a volta não sei se é verdade e se for há como provar?
>>
>> Meu objetivo primário é saber se:
>> 0,123456789112233445566778899111222333444555666777888999... é racional.
>> As reticências se referem ao aumento de mais um algarismo repetido a cada
>> sequência, ou seja a primeira aparição de 1 será 1, a 2a 11 a 3a 111 e
>> assim sucessivamente, o mesmo vale para os demais algarismos.
>>
>> Alguém poderia me ajudar?
>> Grato,
>> PJMS
>>
>> --
>> Esta mensagem foi verificada pelo sistema de antivírus e
>> acredita-se estar livre de perigo.
>
>
> --
> Esta mensagem foi verificada pelo sistema de antivírus e
> acredita-se estar livre de perigo.

-- 
Esta mensagem foi verificada pelo sistema de antiv�rus e
 acredita-se estar livre de perigo.



[obm-l] Re: Dúvida e ajuda.

2022-04-08 Por tôpico Pedro José
Bom dia!
Última forma!
Achei uma demonstração simples e bela, usando casa dos pombos. Uma hora
haverá de ter repetição, portanto, tem que ter um grupamento de dígitos que
se repita caso seja uma série infinita de algarismos decimais.
Portanto o número é irracional.
Grato!
PJMS

Em sex., 8 de abr. de 2022 às 11:06, Pedro José 
escreveu:

> Bom dia!
> Posso concluir que um número representado por uma infinidade de algarismos
> decimais é racional se e somente se tem um período de repetições desses
> algarismos?
> A ida é fácil se tiver o período é racional.
> Já a volta não sei se é verdade e se for há como provar?
>
> Meu objetivo primário é saber se:
> 0,123456789112233445566778899111222333444555666777888999... é racional. As
> reticências se referem ao aumento de mais um algarismo repetido a cada
> sequência, ou seja a primeira aparição de 1 será 1, a 2a 11 a 3a 111 e
> assim sucessivamente, o mesmo vale para os demais algarismos.
>
> Alguém poderia me ajudar?
> Grato,
> PJMS
>

-- 
Esta mensagem foi verificada pelo sistema de antiv�rus e
 acredita-se estar livre de perigo.



[obm-l] Dúvida e ajuda.

2022-04-08 Por tôpico Pedro José
Bom dia!
Posso concluir que um número representado por uma infinidade de algarismos
decimais é racional se e somente se tem um período de repetições desses
algarismos?
A ida é fácil se tiver o período é racional.
Já a volta não sei se é verdade e se for há como provar?

Meu objetivo primário é saber se:
0,123456789112233445566778899111222333444555666777888999... é racional. As
reticências se referem ao aumento de mais um algarismo repetido a cada
sequência, ou seja a primeira aparição de 1 será 1, a 2a 11 a 3a 111 e
assim sucessivamente, o mesmo vale para os demais algarismos.

Alguém poderia me ajudar?
Grato,
PJMS

-- 
Esta mensagem foi verificada pelo sistema de antiv�rus e
 acredita-se estar livre de perigo.



[obm-l] Re: [obm-l] Re: [obm-l] Invertíveis e Divisores de Zero

2021-11-30 Por tôpico Pedro Júnior
Sim...

Em ter., 30 de nov. de 2021 às 15:21, Claudio Buffara <
claudio.buff...@gmail.com> escreveu:

> Z_4 x Z_5 é isomorfo a Z_20.
> Talvez isso ajude.
>
> On Tue, Nov 30, 2021 at 2:33 PM Pedro Júnior 
> wrote:
>
>> Quem puder ajudar...
>> Encontre todos os invertíveis e divisores de zero em Z_4 x Z_5.
>>
>>
>>
>> --
>> Esta mensagem foi verificada pelo sistema de antivírus e
>> acredita-se estar livre de perigo.
>
>
> --
> Esta mensagem foi verificada pelo sistema de antivírus e
> acredita-se estar livre de perigo.



-- 

Pedro Jerônimo S. de O. Júnior

Professor de Matemática

Geo João Pessoa – PB

-- 
Esta mensagem foi verificada pelo sistema de antiv�rus e
 acredita-se estar livre de perigo.



[obm-l] Invertíveis e Divisores de Zero

2021-11-30 Por tôpico Pedro Júnior
Quem puder ajudar...
Encontre todos os invertíveis e divisores de zero em Z_4 x Z_5.

-- 
Esta mensagem foi verificada pelo sistema de antiv�rus e
 acredita-se estar livre de perigo.



[obm-l] Re: [obm-l] Re: [obm-l] Dúvida

2021-11-22 Por tôpico Pedro José
Boa tarde!

Grato, pela ajuda!
Não conheço.
Vou abrir um leque de estudo para tentar entender!
Valeu a curiosidade, com o que cheguei consegui matar o problema.
Genericamente, consegui que a solução levaria a uma expressão que era um
quadrado perfeito,esse era o objetivo. Só que me deu curiosidade, quanto a
resolução. Vou me enveredar no tema.

Cordialmente,
PJMS.

Em ter., 16 de nov. de 2021 às 17:29, Prof. Douglas Oliveira <
profdouglaso.del...@gmail.com> escreveu:

> Equação de Pell
>
> Em seg., 15 de nov. de 2021 13:36, Pedro José 
> escreveu:
>
>> Boa tarde!
>>
>> Alguém saberia como resolver a seguinte equação:
>>
>> x^2-7y^2=1, x,y em Z?
>>
>> Fiz a-7b=1 e achei a= 8 +7k e b=1 +K
>> Logo fica fácil que para k=-1 funciona x^2=1 e y^2=0.
>> Também funciona para k=8 x^2=64 e y^2=9.
>> Mas não sei nem como achar mais soluções nem como provar que só são essas.
>> Alguém poderia me dar uma orientação?
>>
>> Cordialmente,
>> PJMS
>>
>> --
>> Esta mensagem foi verificada pelo sistema de antivírus e
>> acredita-se estar livre de perigo.
>
>
> --
> Esta mensagem foi verificada pelo sistema de antivírus e
> acredita-se estar livre de perigo.

-- 
Esta mensagem foi verificada pelo sistema de antiv�rus e
 acredita-se estar livre de perigo.



[obm-l] Dúvida

2021-11-15 Por tôpico Pedro José
Boa tarde!

Alguém saberia como resolver a seguinte equação:

x^2-7y^2=1, x,y em Z?

Fiz a-7b=1 e achei a= 8 +7k e b=1 +K
Logo fica fácil que para k=-1 funciona x^2=1 e y^2=0.
Também funciona para k=8 x^2=64 e y^2=9.
Mas não sei nem como achar mais soluções nem como provar que só são essas.
Alguém poderia me dar uma orientação?

Cordialmente,
PJMS

-- 
Esta mensagem foi verificada pelo sistema de antiv�rus e
 acredita-se estar livre de perigo.



Re: [obm-l] OBMEP 2021 - Fase 2 - N3

2021-11-09 Por tôpico Pedro Júnior
Obrigado, Ralph!

Em ter., 9 de nov. de 2021 às 13:21, Ralph Costa Teixeira 
escreveu:

> Suponho que (A) e (B) sejam fáceis -- basta seguir o algoritmo na mão e
> ver o que acontece.
>
> Para facilitar a conversa, vou pensar em "tempo" como o número de
> movimentos feitos... Ou seja, o tempo 0 corresponde à posição inicial; o
> tempo 1 seria logo após o primeiro movimento; etc.
>
> Para (C), pense assim: se o sistema tem alguma coroa no tempo (n), eu
> afirmo que vai ter alguma coroa no tempo (n+1). De fato:
> -- Se o ponteiro aponta para uma cara no tempo (n), o sistema não muda, e
> a tal coroa continua ali;
> -- Se o ponteiro aponta para uma coroa no tempo (n), ESTA coroa vai ficar
> presente no tempo (n+1).
> Portanto, sempre teremos coroas.
> (Talvez seja mais natural pensar assim: como que o sistema passaria de
> "ter coroas" para "não ter coroas"? Bom, para ele mudar o ponteiro tem que
> apontar para alguma coroa, e esta coroa NÃO MUDA. Ou seja,
> impossível passar de "ter coroas" para "não ter coroas".)
>
> Para (D), note que o sistema tem apenas (2^10) * 10 configurações
> possíveis (o número não interessa tanto, o que importa é que é FINITO; note
> que incluo ali as posições das moedas E a do ponteiro), enquanto o tempo
> avança sempre, então em algum momento alguma configuração vai ter que
> repetir.
> Mas pense como "desfazer" o último movimento realizado e você vai perceber
> que existe apenas um jeito de "voltar no tempo" (deixo para você descrever
> exatamente isso)! Ou seja, o sistema é reversível (olhando como ficou o
> sistema no tempo (n+1), você consegue deduzir como ele estava no tempo (n),
> revertendo o último movimento, de maneira única). Portanto, se o sistema
> tinha a mesma configuração nos tempos A e A+T, revertendo os movimentos,
> concluímos que vai ter a mesma configuração nos tempos 0 e T; ou seja, no
> tempo T tínhamos todas coroas como no tempo 0 (e o ponteiro apontando para
> A! Bônus!)
>
> Abraço, Ralph.
>
> On Tue, Nov 9, 2021 at 12:22 PM Pedro Júnior 
> wrote:
>
>> Olá pessoal, alguém aí conseguiu fazer essa questão da prova da OBMEP
>> 2021 N3, fase 2? Se puder, ajuda aí... Valeu!
>>
>> 6) há 10 moedas em um círculo nomeadas de A a J, inicialmente todas com a
>> face coroa virada para cima. No centro desse círculo, há um ponteiro que
>> inicialmente aponta para a moeda A. Esse ponteiro se movimenta, girando no
>> sentido anti-horário (A->B->C->...->J->A->...). Ao movimentar-se, há duas
>> opções:
>> •Quando o ponteiro termina o movimento apontando para uma moeda com a
>> face coroa virada para cima, a moeda seguinte é virada.
>> •Quando o ponteiro termina o movimento apontando para uma moeda com a
>> face cara virada para cima, nada acontece.
>>
>> Há exemplo, no primeiro movimento (de A para B), o ponteiro termina em B,
>> e assim, vira-se a moeda C, que fica com a face cara para cima.
>>
>> Letra A) o que acontece com as moedas C e D após o segundo movimento?
>>
>> Letra B) Depois do 12º movimento, quais moedas estão com a face coroa
>> virada para cima?
>>
>> Letra C) mostre que é impossível que, após certo número de movimentos,
>> todas as moedas fiquem com a face cara para cima.
>>
>> Letra D) Mostre que, após um certo número de movimentos, todas as moedas
>> voltarão a ficar com a face coroa para cima.
>>
>>
>>
>> --
>> Esta mensagem foi verificada pelo sistema de antivírus e
>> acredita-se estar livre de perigo.
>
>
> --
> Esta mensagem foi verificada pelo sistema de antivírus e
> acredita-se estar livre de perigo.



-- 

Pedro Jerônimo S. de O. Júnior

Professor de Matemática

Geo João Pessoa – PB

-- 
Esta mensagem foi verificada pelo sistema de antiv�rus e
 acredita-se estar livre de perigo.



[obm-l] OBMEP 2021 - Fase 2 - N3

2021-11-09 Por tôpico Pedro Júnior
Olá pessoal, alguém aí conseguiu fazer essa questão da prova da OBMEP 2021
N3, fase 2? Se puder, ajuda aí... Valeu!

6) há 10 moedas em um círculo nomeadas de A a J, inicialmente todas com a
face coroa virada para cima. No centro desse círculo, há um ponteiro que
inicialmente aponta para a moeda A. Esse ponteiro se movimenta, girando no
sentido anti-horário (A->B->C->...->J->A->...). Ao movimentar-se, há duas
opções:
•Quando o ponteiro termina o movimento apontando para uma moeda com a face
coroa virada para cima, a moeda seguinte é virada.
•Quando o ponteiro termina o movimento apontando para uma moeda com a face
cara virada para cima, nada acontece.

Há exemplo, no primeiro movimento (de A para B), o ponteiro termina em B, e
assim, vira-se a moeda C, que fica com a face cara para cima.

Letra A) o que acontece com as moedas C e D após o segundo movimento?

Letra B) Depois do 12º movimento, quais moedas estão com a face coroa
virada para cima?

Letra C) mostre que é impossível que, após certo número de movimentos,
todas as moedas fiquem com a face cara para cima.

Letra D) Mostre que, após um certo número de movimentos, todas as moedas
voltarão a ficar com a face coroa para cima.

-- 
Esta mensagem foi verificada pelo sistema de antiv�rus e
 acredita-se estar livre de perigo.



[obm-l] Re: [obm-l] cálculo

2021-09-15 Por tôpico Pedro Angelo
A definição de integrabilidade Riemann passa por verificar que, para
partições P suficientemente finas, a soma superior S(f;P) é parecida
com a soma inferior s(f;P).

Faça o que sempre deve ser feito nesse tipo de problema: calcule
exemplos concretos. Escolha partições quaisquer (pequenas, pois vc
quer conseguir fazer as contas na mão), e calcule as somas inferior e
superior para cada partição escolhida. O que acontece à medida que as
partições vão ficando cada vez mais finas?

On Wed, Sep 15, 2021 at 12:11 AM Israel Meireles Chrisostomo
 wrote:
>
> Olá pessoal. eu estou me esforçando para entender esse exemplo do guidorizzi, 
> alguém poderia me explicar?Aqui vai:
> Seja f uma função, tal que se x é racional então f igual a 1, se x é 
> irracional então f igual a zero. Mostre que a função não é riemann integrável.
>
> --
> Israel Meireles Chrisostomo
>
> --
> Esta mensagem foi verificada pelo sistema de antivírus e
> acredita-se estar livre de perigo.

-- 
Esta mensagem foi verificada pelo sistema de antiv�rus e
 acredita-se estar livre de perigo.


=
Instru��es para entrar na lista, sair da lista e usar a lista em
http://www.mat.puc-rio.br/~obmlistas/obm-l.html
=


[obm-l] Re: [obm-l] Ajuda numa questão da OBM 1987

2021-07-20 Por tôpico joao pedro b menezes
Eu pensaria em trabalhar com os pontos notáveis, talvez o baricentro, e
argumentar que em qualquer outro ponto é possível realizar um corte que o
prejudique mais. Isso é só uma teoria e, portanto, é possível que esteja
totalmente errada.

-- 
Esta mensagem foi verificada pelo sistema de antiv�rus e
 acredita-se estar livre de perigo.



[obm-l] f(x + y) = f(x) + f(y)

2021-05-05 Por tôpico joao pedro b menezes
Eu estava fazendo um exercício de equações funcionais e me deparei com essa
expressão. Não sei o que aconteceu, mas tive uma crise existencial e decidi
provar que isso implica f(x) = ax + b( ou pelo menos acho que implica).
Essa prova estaria certa?:
(obs: a função é definida nos racionais)
f(x + 0) = f(x) + f(0) => f(0) = 0.
f(x + h) = f(x) + f(h) ->
(f(x + h) - f(x))/h = f(h)/h = (f(h) - f(0))/h
agora basta fazer lim h -> 0 e obtemos
f’(x) = f’(0) . Mas f’(0) é uma constante, logo f(x) = ax + b
(obs: tenho quase certeza que ela seria válida para os reais, porém como a
função é limitada aos racionais, estou em dúvida)

-- 
Esta mensagem foi verificada pelo sistema de antiv�rus e
 acredita-se estar livre de perigo.



Re: [obm-l] Magnitude

2021-04-25 Por tôpico Pedro Lazéra
Boa noite, Maikel.

A quantidade de algarismos de um número x (na base 10) é "1 + piso de
log(x)", em que "log" é a função logaritmo na base 10. Você pode verificar
isso assim: 10^n, para n inteiro >= 0, é o menor número do mundo com n+1
algarismos. Além disso, log(10^n) = n. Por fim, log é uma função crescente.

Usando que log(a*b) = log(a) + log(b), uma solução computeira é fazer um
programa que calcula log(i), para i inteiro entre 1 e 100, e soma esses
valores a uma variável s inicializada com o valor ZERO.

Uma solução computeira (provavelmente) errada é calcular x = 100! e depois
achar o log(x). Esse valor x não cabe nas estruturas de dados que a maioria
das linguagens usa para representar números.

Bom, a soma dá 157.97, conforme o Anderson Torres falou antes de mim. A
gente sabe que os computadores até erram (truncam) o valor exato de log(x),
mas o erro é bem pequeno e só estamos somando 100 aplicações da função log,
daí sabemos que esse 157.97 pode até estar errado, mas é por muito pouco
(menos do que 0,01, por exemplo).

Finalmente, 100! tem 1 + piso(157.97) = 158 algarismos.

Abraços,
Pedro

On Sun, Apr 11, 2021 at 12:29 AM Anderson Torres <
torres.anderson...@gmail.com> wrote:

> Em sáb., 3 de abr. de 2021 às 01:13, Maikel Andril Marcelino
>  escreveu:
> >
> > Quantos algarismos tem o número (100!) ?
>
> Em outras palavras, qual é o log(100!)/log(10). O Google me diz que
> isso é 157,97 - logo, 158 dígitos.
>
> >
> >
> > Atenciosamente,
> >
> > Maikel Andril Marcelino
> > Assistente de Aluno - Biblioteca - Ramal: 7616
> > Coordenadoria de Apoio Acadêmico - COAPAC/IFRN-SPP
> > Instituto Federal do Rio Grande do Norte
> > Campus São Paulo do Potengi
> >
> > +55 (84) 8851-3451
> >
> > --
> > Esta mensagem foi verificada pelo sistema de antivírus e
> > acredita-se estar livre de perigo.
>
> --
> Esta mensagem foi verificada pelo sistema de antivírus e
>  acredita-se estar livre de perigo.
>
>
> =
> Instru�ões para entrar na lista, sair da lista e usar a lista em
> http://www.mat.puc-rio.br/~obmlistas/obm-l.html
> =
>

-- 
Esta mensagem foi verificada pelo sistema de antiv�rus e
 acredita-se estar livre de perigo.



[obm-l] Re: [obm-l] RES: [obm-l] Re: [obm-l] INFLAÇÂO MÁXIMA

2021-04-24 Por tôpico Pedro Júnior
Sobre a gramática, verdade!
Sobre a Matemática, impecável.
Consegui ver onde eu estava errando.

Obrigado, professor.

Em sex, 23 de abr de 2021 14:24,  escreveu:

> Olá!
>
> Para começar, esta questão deveria ter sido anulada. “… não HAJAM perdas
> reais?” é um assassinato da nossa língua.
>
>
>
> Juros “reais” (JR), de 10%, significam juros acima da inflação (IF).
>
> No período de 1 ano, o ganho bruto de capital (GB) será: GB = 1.000
> (1+10%)(1+IF) - 1.000
>
> Descontando o imposto, o ganho líquido (GL) será: GL =  (1-40%)GB
>
> Condição de contorno: não “hão” (coerência com o linguajar da questão)
> perdas reais: 1.000(1+IF) = 1.000+GL = 1.000 + (1-40%)GB = 1.000 + (1-40%)(
> 1.000 (1+10%)(1+IF) - 1.000 )
>
> Daí: IF = 17,6470…%
>
>
>
> *Albert Bouskelá*
>
> bousk...@gmail.com
>
>
>
> *De:* owner-ob...@mat.puc-rio.br  *Em nome de
> *Daniel Jelin
> *Enviada em:* sexta-feira, 23 de abril de 2021 12:30
> *Para:* obm-l@mat.puc-rio.br
> *Assunto:* [obm-l] Re: [obm-l] INFLAÇÂO MÁXIMA
>
>
>
> Curioso, pra mim deu muito perto, 17,6470...%
>
> Resolvi a seguinte inequação, com x = 1 + (inflação):
>
> 1.1*1000x - (1.1*1000x - 1000)*0.4>=1000x
> 1.1 x - 0.44 x + 0.4 >= x
> x<=0.4/0.34= 1.176470...
>
> Parece simples. O que tá escapando aqui?
>
>
>
> On Fri, Apr 23, 2021 at 11:23 AM Pedro Júnior 
> wrote:
>
> Olá pessoal, acabei me enrolando nesse probleminha da Olimpíada Brasileira
> de Economia. Será que alguém pode me ajudar? Vai junto o gabarito da
> competição, isso foi em 2020.
>
>
>
> *01)* Um título comprado por mil reais promete pagar juros reais de 10%
> a.a. A alíquota de imposto é de 40%. Qual a inflação máxima no período para
> que não hajam perdas reais?
>
> Resp.: 17,62%
>
>
>
> --
>
> Pedro Jerônimo S. de O. Júnior
>
> Professor de Matemática
>
>
>
>
> --
> Esta mensagem foi verificada pelo sistema de antivírus e
> acredita-se estar livre de perigo.
>
>
> --
> Esta mensagem foi verificada pelo sistema de antiv?s e
> acredita-se estar livre de perigo.
>
> --
> Esta mensagem foi verificada pelo sistema de antivírus e
> acredita-se estar livre de perigo.
>

-- 
Esta mensagem foi verificada pelo sistema de antiv�rus e
 acredita-se estar livre de perigo.



[obm-l] INFLAÇÂO MÁXIMA

2021-04-23 Por tôpico Pedro Júnior
Olá pessoal, acabei me enrolando nesse probleminha da Olimpíada Brasileira
de Economia. Será que alguém pode me ajudar? Vai junto o gabarito da
competição, isso foi em 2020.

*01)* Um título comprado por mil reais promete pagar juros reais de 10%
a.a. A alíquota de imposto é de 40%. Qual a inflação máxima no período para
que não hajam perdas reais?
Resp.: 17,62%

-- 

Pedro Jerônimo S. de O. Júnior

Professor de Matemática

-- 
Esta mensagem foi verificada pelo sistema de antiv�rus e
 acredita-se estar livre de perigo.



Re: [obm-l] Artigo

2021-04-02 Por tôpico Pedro Júnior
Boa discussão!

Em ter, 30 de mar de 2021 17:16, Israel Meireles Chrisostomo <
israelmchrisost...@gmail.com> escreveu:

> Obrigado
>
> Em ter., 30 de mar. de 2021 às 16:20, Daniel Jelin 
> escreveu:
>
>> não sei ao certo, meu caro, mas, falando como professor (e leitor),
>> suponho que não. e não é tanto por ser muito ou pouco avançado. receio que
>> o assunto fuja às preocupações do ensino básico - mesmo que a sua prova
>> seja elementar. repara, nada contra provas matemáticas na escola, ao
>> contrário. acho importante mostrar para os alunos de onde vêm os teoremas,
>> claro, mas: apenas das propriedades que eles de fato usam; e apenas as
>> demonstrações que eles têm condição de acompanhar do princípio ao fim, sem
>> que isso se torne um fardo adicional. será o caso?
>>
>> claro, algumas provas podem interessar ao professor mesmo que ele não
>> tenha a intenção de levá-la a todos os alunos. a irracionalidade de pi,
>> talvez, pra ficar no mesmo campo. leria com gosto uma investigação sobre a
>> irracionalidade de pi, passo a passo, com as armas da matemática do ensino
>> médio. tem isso? sei lá eu. mas a transcendência de pi? que tipo de
>> questão, que problema (escolar) esbarra na transcendência de pi? uma
>> sugestão: se vc puder mostrar que o professor deveria, sim, se importar com
>> isso, que questões importantes passam por aí, opa, então beleza, aí fica
>> mto legal, aí tem tudo a ver.
>>
>> abs
>>
>> On Tue, Mar 30, 2021 at 2:14 PM Israel Meireles Chrisostomo <
>> israelmchrisost...@gmail.com> wrote:
>>
>>> Vcs acham que a revista RPM aceitaria uma prova para transcendência de
>>> pi, ou isso é algo avançado demais para revista?
>>>
>>> --
>>> Israel Meireles Chrisostomo
>>>
>>> --
>>> Esta mensagem foi verificada pelo sistema de antivírus e
>>> acredita-se estar livre de perigo.
>>
>>
>> --
>> Esta mensagem foi verificada pelo sistema de antivírus e
>> acredita-se estar livre de perigo.
>
>
>
> --
> Israel Meireles Chrisostomo
>
> --
> Esta mensagem foi verificada pelo sistema de antivírus e
> acredita-se estar livre de perigo.

-- 
Esta mensagem foi verificada pelo sistema de antiv�rus e
 acredita-se estar livre de perigo.



[obm-l] Re: [obm-l] Re: [obm-l] Re: [obm-l] Re: [obm-l] Re: [obm-l] Re: [obm-l] Re: Equações funcionais

2021-02-16 Por tôpico joao pedro b menezes
Eu sei, temos f(-1)= 0, f(0) = 1, e f é bijetora. Após trabalhar a equação
que cheguei na expressão:
f( x + f(x) ) - f( f(x)) = x.  Queria saber se essa identidade, junto com a
do enunciado, é suficiente para provar a linearidade de f.

-- 
Esta mensagem foi verificada pelo sistema de antiv�rus e
 acredita-se estar livre de perigo.



[obm-l] Re: [obm-l] Re: [obm-l] Re: [obm-l] Re: [obm-l] Re: Equações funcionais

2021-02-16 Por tôpico joao pedro b menezes
Foi da OBM 2006, nível 3,  3° fase:
“Determine todas as funções f: R -> R tais que
f( xf(y) + f(x) ) = 2f(x) + xy
para todos x,y reais”

-- 
Esta mensagem foi verificada pelo sistema de antiv�rus e
 acredita-se estar livre de perigo.



[obm-l] Re: [obm-l] Re: [obm-l] Re: Equações funcionais

2021-02-14 Por tôpico joao pedro b menezes
Obrigado pela resposta, mas ainda tenho umas dúvidas. Poderia dar um
exemplo de tal função ou explicar como construí-la? E se f fosse somente
injetora, mudaria alguma coisa?

-- 
Esta mensagem foi verificada pelo sistema de antiv�rus e
 acredita-se estar livre de perigo.



[obm-l] Re: Equações funcionais

2021-02-14 Por tôpico joao pedro b menezes
Obs: f é bijetora

>

-- 
Esta mensagem foi verificada pelo sistema de antiv�rus e
 acredita-se estar livre de perigo.



[obm-l] Equações funcionais

2021-02-14 Por tôpico joao pedro b menezes
Olá, bom dia. Eu estava fazendo um exercício de equações funcionais e
acabei concluindo que :
f( f(x) + x ) - f( f( x) ) = x para todo x real. Somente isso é suficiente
para provar que f é linear?

-- 
Esta mensagem foi verificada pelo sistema de antiv�rus e
 acredita-se estar livre de perigo.



[obm-l] Re: [obm-l] Indução

2021-02-05 Por tôpico joao pedro b menezes
obs: só agora fui ver o título :) , se era necessário fazer especialmente
por indução, por favor desconsidere a minha resposta.

On Fri, Feb 5, 2021 at 7:14 AM joao pedro b menezes <
joaopedrobmene...@gmail.com> wrote:

> Suponha que d | (a^(2)^n ) + 1. Então a^2^n = -1 (mod d). Pegue um primo
> tal que p| d, então a^2^n = -1 (mod p). Mas temos: a^2^(n+1) = 1 (mod p).
> Logo
>  ord(p)a | 2^(n+1), mas ord(p)a não divide 2^n, logo ord(p)a = 2^(n + 1).
> Isso é um absurdo, pois ord(p)a < p <= d <= 2^(n + 1).
> obs: tenho quase certeza que já perguntaram a mesma coisa nessa lista.
> Portanto acho que vale a pena ir procurar a resposta anterior também :)
>
> On Thu, Feb 4, 2021 at 11:20 PM Heitor Gama Ribeiro 
> wrote:
>
>> Prove por indução que se 3<= d <= 2^(n+1), então d não divide
>> [a^(2)^(n) + 1] para todo inteiro positivo a.
>>
>>
>> Sent from my iPhone
>>
>> =
>> Instruções para entrar na lista, sair da lista e usar a lista em
>> http://www.mat.puc-rio.br/~obmlistas/obm-l.html
>> =
>>
>


[obm-l] Re: [obm-l] Indução

2021-02-05 Por tôpico joao pedro b menezes
Suponha que d | (a^(2)^n ) + 1. Então a^2^n = -1 (mod d). Pegue um primo
tal que p| d, então a^2^n = -1 (mod p). Mas temos: a^2^(n+1) = 1 (mod p).
Logo
 ord(p)a | 2^(n+1), mas ord(p)a não divide 2^n, logo ord(p)a = 2^(n + 1).
Isso é um absurdo, pois ord(p)a < p <= d <= 2^(n + 1).
obs: tenho quase certeza que já perguntaram a mesma coisa nessa lista.
Portanto acho que vale a pena ir procurar a resposta anterior também :)

On Thu, Feb 4, 2021 at 11:20 PM Heitor Gama Ribeiro 
wrote:

> Prove por indução que se 3<= d <= 2^(n+1), então d não divide
> [a^(2)^(n) + 1] para todo inteiro positivo a.
>
>
> Sent from my iPhone
>
> =
> Instruções para entrar na lista, sair da lista e usar a lista em
> http://www.mat.puc-rio.br/~obmlistas/obm-l.html
> =
>


[obm-l] Re: [obm-l] Re: [obm-l] Função parte inteira

2021-02-03 Por tôpico joao pedro b menezes
Obrigado pela dica! Honestamente creio que existe um erro nesse problema.
Fazendo alguns casos na mão é possivel perceber que isso sempre resulta em
8n + 7. Essa é a prova:
"Provar que ( n^(1/3) + ( n + 2)^(1/3) )³  < 8n + 8. Abrindo a potência,
temos:
2n + 2 + 3 * ( (n² ( n + 2))^(1/3) + (n(n + 2)²)^(1/3)) < 8n + 8
  (n² ( n + 2))^(1/3) + (n(n + 2)²)^(1/3)   < 2n + 2
Porém temos que  (n² ( n + 2))^(1/3) < n + 2/3  , e  (n(n + 2)²)^(1/3) <
n + 4/3 ( eu testei elevando ambos os lados ao cubo deu certo) . Isso
confirma a inequação inicial.
Agora se 8n + 7 <  ( n^(1/3) + ( n + 2)^(1/3) )³  o exercício acaba. De
fato, trabalhando a expressão:
   (n² ( n + 2))^(1/3) + (n(n + 2)²)^(1/3)   > 2n + 5/3
Mas novamente, tem se que  (n² ( n + 2))^(1/3) > n + 1/2 e  (n(n +
2)²)^(1/3) > n + 7/6 para qualquer n > 1 ( no caso n =1 basta testar na
mão). E como 1/2 + 7/6 = 5/3 ,  tem se que ela é verdade, logo:
8n + 7 <  ( n^(1/3) + ( n + 2)^(1/3) )³  < 8n + 8 ==> [ ( n^(1/3) + ( n +
2)^(1/3) )³ ] = 8n + 7"
Eu estranhei bastante porque nunca tinha acontecido de um exercicio do POTI
estar errado.
obs: Se a minha solução estiver errada de alguma forma, adoraria saber!

On Wed, Feb 3, 2021 at 12:42 PM Ralph Costa Teixeira 
wrote:

> Sem tempo agora, mas olhando por alto eu aproximaria o que estah dentro do
> () por 2(n+1)^(1/3), o que levaria imediatamente a 8(n+1). Serah que a
> parte inteira daquela coisa eh 8(n+1)?
>
> Entao eu tentaria abrir os cubos, subtrair 8(n+1), e mostrar que o que
> sobra eh menor que 1.
>
> Serah que funciona?
>
> On Wed, Feb 3, 2021 at 10:03 AM joao pedro b menezes <
> joaopedrobmene...@gmail.com> wrote:
>
>> Olá, estava tentando fazer esta questão:
>>   Prove que [ ( n^(1/3) + (n + 2)^(1/3)  )³] é divisível por 8.
>> obs: não tinha a tecla de função parte inteira, por isso escolhi [ ]
>> Se alguém tiver alguma dica ou souber como resolver, ajudaria bastante.
>>
>


[obm-l] Função parte inteira

2021-02-03 Por tôpico joao pedro b menezes
Olá, estava tentando fazer esta questão:
  Prove que [ ( n^(1/3) + (n + 2)^(1/3)  )³] é divisível por 8.
obs: não tinha a tecla de função parte inteira, por isso escolhi [ ]
Se alguém tiver alguma dica ou souber como resolver, ajudaria bastante.


Re: [obm-l] Limites

2021-01-30 Por tôpico joao pedro b menezes
Olá, boa noite, obrigado pela resposta( e pela dica)! Quanto ao meu
conhecimento de cálculo, embora saiba um pouco, ele é limitado e portanto
não conhecia esse teorema. Já li sobre ele durante o dia e entendi sua
demonstração. Mais uma vez, obrigado aos dois!


Re: [obm-l] Limites

2021-01-29 Por tôpico Pedro Angelo
Em geral, sempre que você não sabe o que fazer com uma potência (por
exemplo nesse caso em que tanto a base quanto o expoente dependem de
x), a dica é trocar a base B por e^(log(B)).

Trocando (1+x) por e^(log(1+x)), vai ficar:

e^( ln(1+x) / x )

Como a função f(u)=e^u é contínua, basta saber calcular o limite de:

ln(1+x) / x

quando x tende a infinito. Esse é mais fácil?

On Fri, Jan 29, 2021 at 10:26 PM joao pedro b menezes
 wrote:
>
> Olá a todos, boa noite, estou com um pouco de dificuldade em encontrar uma 
> prova para esse limite
> lim x-> infinito (1 + x)^(1/x)
> Creio que ele seja 1, mas não conheço nenhuma maneira de provar isso
> Se alguém tiver uma dica ou souber como provar, ajudaria bastante
> Já agradeço pela ajuda :)

=
Instru��es para entrar na lista, sair da lista e usar a lista em
http://www.mat.puc-rio.br/~obmlistas/obm-l.html
=


[obm-l] Limites

2021-01-29 Por tôpico joao pedro b menezes
Olá a todos, boa noite, estou com um pouco de dificuldade em encontrar uma
prova para esse limite
lim x-> infinito (1 + x)^(1/x)
Creio que ele seja 1, mas não conheço nenhuma maneira de provar isso
Se alguém tiver uma dica ou souber como provar, ajudaria bastante
Já agradeço pela ajuda :)


[obm-l] Proxima OBM

2020-12-19 Por tôpico joao pedro b menezes
Olá, bom dia. Meu nome é João Pedro Menezes. Eu contactei vocês à um tempo
atrás para saber quando seria liberada a lista de convidados para a OBM
2020 (que agora será em 2021), mas obtive uma resposta inconclusiva. se
puderem me ajudar, agradeceria muito.

João Pedro Menezes


[obm-l] Re: [obm-l] Re: [obm-l] Re: [obm-l] Ângulos de um triângulo

2020-12-04 Por tôpico Pedro Henrique de Almeida Ursino
Seja x a medida do ângulo DAC (logo DAB mede 48 -x). Por trig Ceva

sin x * sin 18 * sin 54 = sin (48-x) * sin 12 * sin 48.

Pode-se deduzir que sin 54 = (1+ sqrt(5))/4 e sin 18 = (sqrt(5)-1)/4. Logo,
sin 54 * sin 18 = 1/4. Assim, nossa equação fica

sin x / sin (48-x) = 4 * sin 12 * sin 48

usando Werner, temos que 2 *sin 12 * sin 48 = cos(48 -12) - cos (48 + 12) =
cos 36 - cos 60 = cos 36 - 1/2
Desse modo, nossa relação fica

sinx/ sin(48-x) = 2*cos 36 - 1 = 2*sin 18

Daí é muito trivial ver que 18 é solução. Esta solução é única pois f(x) =
sin x/ sin(48-x) é crescente para x entre 0 e 48 (graus).

Essa solução usa muitas relações trigonométricas não tão conhecidas
assim Essa foi uma das soluções dadas pelo prof. Sandro do Canal: A
hora do Bizu em seu último vídeo. Ele também dá uma solução sintética para
o problema. Enfim, vale a pena conferir.

On Fri, Dec 4, 2020 at 4:13 PM Armando Staib 
wrote:

> Não querendo polemizar, mas de acordo com o exercício, é, na minha
> opinião, impossível ser 30 o ângulo pedido  pq se fosse o triângulo DBC
> teria o lado oposto ao ângulo de 18 menor do que o lado oposto ao ângulo de
> 12.
>
> Se me enganei poderiam me mostrar, onde eu errei?
>
> Em sex., 4 de dez. de 2020 às 14:06, Claudio Buffara <
> claudio.buff...@gmail.com> escreveu:
>
>> Aliás, de posse da expressão para BAD e CAD, um exercício razoavelmente
>> fácil de programação (até em planilha), é descobrir para quais triângulos
>> isósceles com ângulos inteiros (em graus) e quais ângulos DBC e DCB
>> inteiros, BAD (e obviamente CAD) também são inteiros.
>>
>> Daí, um problema (não mais um exercício!) é descobrir o padrão por trás
>> destes triângulos especiais.
>>
>> On Fri, Dec 4, 2020 at 1:42 PM Claudio Buffara 
>> wrote:
>>
>>> Usando áreas - em particular, área(ABC) = (1/2)*AB*AC*sen(A) - você
>>> consegue, com alguma facilidade, expressar a tangente de DAC em termos de
>>> senos e cossenos dos ângulos dados.   Daí, é só calcular (com calculadora
>>> ou computador - eu uso Excel ou Wolfram Alpha).  E, de fato, AD divide BAC,
>>> que mede 48 graus, em dois ângulos: um medindo 30 e o outro 18 graus.
>>>
>>> O que não dá é - em 2020 - ficar manipulando aquelas fórmulas de
>>> prostaférese ou identidades trigonométricas obscuras envolvendo ângulos
>>> múltiplos de 3 graus. Isso é coisa do século 19...
>>>
>>> []s,
>>> Claudio.
>>>
>>> On Mon, Nov 30, 2020 at 7:28 PM Professor Vanderlei Nemitz <
>>> vanderma...@gmail.com> wrote:
>>>
 Boa noite!
 Alguém conhece uma saída para o seguinte problema?
 Muito obrigado!

 *Num triângulo isósceles ABC, AB = AC.*
 *Seja D um ponto interno tal que os ângulos DBC, DCB, DBA e DCA medem,
 respectivamente, 12°, 18°, 54° e 48°. *
 *Determine a medida do ângulo DAC.*


 
  Livre
 de vírus. www.avast.com
 .

 <#m_-9147473276317047412_m_6354815198100344298_m_-4747407596740689255_m_4608836649714424769_DAB4FAD8-2DD7-40BB-A1B8-4E2AA1F9FDF2>

>>>


[obm-l] Teoria dos Números

2020-11-20 Por tôpico Pedro Henrique de Almeida Ursino
A problema que segue é o problema 8 da primeira lista de preparação para a
Cone Sul/OMCPLP do ano de 2020. Segue o problema:


Para cada inteiro positivo n, defina
  S_n = 1!+2!+...+n!
Prove que existe um inteiro positivo n tal que S_n possui um divisor primo
maior que 10^(2020)



Espero que gostem do problema!


[obm-l] Probabilidade - duas listas a partir da normal(0,1)

2020-11-11 Por tôpico Pedro Lazéra


[obm-l] Teorema Chinês do resto

2020-10-27 Por tôpico joao pedro b menezes
Olá, eu estava fazendo esse exercício :
" . (OBM 2005) Dados os inteiros positivos a, c e o inteiro b, prove que
existe um inteiro positivo x tal que a^x + x ≡ b (mod c)."

Eu pensei nessa solução, mas eu tenho quase certeza que ela está errada...

"Primeiramente , suponhamos c primo. Desse modo, se escolhermos x tal que
x  ≡ 0 (mod c - 1) , teremos a^x + x  ≡ 1 + x <=> x  ≡ b - 1 (mod c) (pelo
teorema de fermat) . Teríamos o sistema de congruências:
x  ≡ 0 ( mod c)
x  ≡ b - 1 (mod c-1)
Como c e c-1 são primos entre sí, pelo teorema chinês do resto esse sistema
infinitas soluções.
Agora, suponhamos c composto. Como um número composto é nada mais que o
produto de uma quantidade finita de primos, podemos chamar todos os primos
divisores de c como p1, p2, p3 ... pn . Forçando  x  ≡ b - 1 (mod pi) para
qualquer pi divisor primo de c, montamos o sistema de congruências:
x  ≡ 0 (mod p1 - 1)
x  ≡ b - 1 (mod p1)
.
x  ≡ 0 (mod pn - 1)
x  ≡ b - 1 (mod pn)
O único empecilho para o teorema é que pj - 1 e ph - 1 ( com j e h inteiros
1 <= j <= h <= n) possivelmente terão múltiplos em comum. Para anular esse
problema, basta fazer com que x seja múltiplo de p1 - 1, p2 - 1 . pn -
1,e chamando de Z o produto de todos esses números, podemos construir:
x  ≡ b - 1 (mod p1)
x  ≡ b - 1( mod p2)

x  ≡ b - 1 (mod pn)
x  ≡ 0 (mod Z)
Como p1, p2 , ... pn e  Z são primos entre si, o sistema sempre terá
infinitas soluções pelo teorema chinês do resto
Dessa forma, comprovamos o enunciado"

Se ela estiver errada( o que eu tenho quase certeza) , alguém poderia, por
favor, me falar por que ?
Agradeço pela ajuda e pelo tempo por ler este email gigante 


[obm-l] Re: [obm-l] Re: [obm-l] Teoria dos Números

2020-10-26 Por tôpico joao pedro b menezes
Muito obrigado pela ajuda! Entendi o exercício agora.

Em dom, 25 de out de 2020 às 19:59, Otávio Araújo 
escreveu:

> Vc resolve essa questão mostrando q p=n^2+n+1.  Se n=1 acabou. Se n>1,Já
> que p divide n^3-1 e é primo, temos que p divide n-1 ou n^2+n+1. Não
> podemos ter p dividindo n-1 pois n divide p-1 -> n<= p-1 n-1 Portanto p divide n^2+n+1. Faca n^2+n+1 = kp, k inteiro positivo. Temos que
> kp=n^2+n+1 é congruente a 1 módulo n. Do enunciado temos p congruente a 1
> módulo n,  mas p é congruente a 1 módulo n e é diferente de 1(pois é primo)
> -> p>= n+1 e  k será congruente a 1 módulo n também. Suponha que k>1,
> k>1 implica k>= n+1 daí kp>=(n+1)^2 > n^2+n+1, contradição. Portanto  k=1
> e p=n^2+n+1.
>
> Em dom, 25 de out de 2020 17:37, joao pedro b menezes <
> joaopedrobmene...@gmail.com> escreveu:
>
>> Olá, boa tarde.
>> Estou com dúvida nesse exercício:
>> " Sejam n um inteiro positivo maior que 1 e p um primo positivo tal que n
>> divide p − 1 e p divide n 3 − 1. Mostre que 4p − 3 ´e um quadrado perfeito."
>> Já agradeço pela ajuda e pelo tempo!
>>
>>


[obm-l] Teoria dos Números

2020-10-25 Por tôpico joao pedro b menezes
Olá, boa tarde.
Estou com dúvida nesse exercício:
" Sejam n um inteiro positivo maior que 1 e p um primo positivo tal que n
divide p − 1 e p divide n 3 − 1. Mostre que 4p − 3 ´e um quadrado perfeito."
Já agradeço pela ajuda e pelo tempo!


Re: [obm-l] Lista/ Livros Geometria IMO/OBM

2020-10-25 Por tôpico joao pedro b menezes
Olá, boa tarde. Eu não conheço todos, mas eu sei que é possivel entrar no
site da OBM :
https://www.obm.org.br/2020/07/25/conheca-livros-para-iniciar-a-preparacao-para-a-proxima-obm/

Ainda assim, um livro que eu particularmente acho fantástico se chama
“Challenging  problems in geometry “. Ele é usado para a preparação da IMO.


[obm-l] Re: [obm-l] Re: [obm-l] Re: [obm-l] Re: [obm-l] Ajuda em teoria dos números

2020-10-22 Por tôpico Pedro José
Boa tarde!
Na verdade: 2^a=64; a= 6 e y=12.

Em qui., 22 de out. de 2020 às 11:17, Pedro José 
escreveu:

> Bom dia!
> Recebi esse problema hoje: 615 + x^2 = 2^y., para x,y inteiros Não saberia
> fazer, como não soube resolver esse, acima. Mas devido a solução do colega
> Esdras, pensei:"já vi algo parecido".
> Basta restringir y aos pares.
> Se y é ímpar x^2=2 mod3, absurdo então y é par. Logo y=2a, com a inteiro.
> (2^a + x) (2^a-x)= 615= 1*615=3*205=5*123=15*41 e como a soma dos fatores
> necessita ser uma potência de 2, só serve para 123 e 5.
> Logo 2^y=64 e y=6 e x= 59 ou x=-59.
> Uma resolução levou a outra, não pelo talento nato, mas por aprendizado, o
> que é válido.
> Teve uma feita que estava tentando provar que o triângulo órtico, era o
> triângulo de menor perímetro que poderia ser inscrito em um triângulo
> acutângulo. Tentei por geometria analítica e só levando tinta. Tinha
> desistido. Quando me deparei com um problema que não consegui resolver, mas
> que tinha um caminho para a solução. Quando vi o rebatimento feito, pensei
> está resolvido. O curioso, é que, quando desisti, pesquisei na internet e
> não achei nada. Depois que consegui resolver, achei duas soluções, e
> infelizmente e como esperado, a minha não era novidade, era clássica.
> Obrigado, Esdras! Sem a sua solução, certamente, não teria resolvido essa
> última questão.
>
> Cordialmente,
> PJMS
>
> Em sex., 24 de jul. de 2020 às 12:19, Prof. Douglas Oliveira <
> profdouglaso.del...@gmail.com> escreveu:
>
>> Obrigado Claudio e Esdras, fatoração show
>>
>>
>> Em sex., 24 de jul. de 2020 às 11:12, Esdras Muniz <
>> esdrasmunizm...@gmail.com> escreveu:
>>
>>> Se for solução inteira positiva, acho que só tem 3 e 4. Vc supõe spdg x
>>> maior ou igual a y, vê que y=1 não tem solução e x=y tb não. Daí, x>y>1.
>>> Fatorando a expressão, fica: (xy-8-(x-y))(xy-8+(x-y))=15. Como
>>> (xy-8-(x-y))>(xy-8+(x-y))>-2. Temos que ou (xy-8-(x-y))=1 e (xy-8+(x-y))=15,
>>> o que não tem soluções inteiras positivas, ou (xy-8-(x-y))=3 e 
>>> (xy-8+(x-y))=5,
>>> cujas únicas soluções inteiras são x=4 e y=3.
>>>
>>> Em sex, 24 de jul de 2020 10:36, Claudio Buffara <
>>> claudio.buff...@gmail.com> escreveu:
>>>
>>>> Pelo que entendi, a solução é a porção dessa curva algébrica situada no
>>>> 1o quadrante.
>>>> Dá pra fazer isso no Wolfram Alpha, com o comando plot (x*y-7)^2 - x^2
>>>> - y^2 = 0.
>>>>
>>>> []s,
>>>> Claudio.
>>>>
>>>> On Fri, Jul 24, 2020 at 9:58 AM Prof. Douglas Oliveira <
>>>> profdouglaso.del...@gmail.com> wrote:
>>>>
>>>>> Preciso de ajuda para encontrar todas as soluções não negativas da
>>>>> equação
>>>>> (xy-7)^2=x^2+y^2.
>>>>>
>>>>> Desde já agradeço a ajuda
>>>>> Douglas Oliveira
>>>>>
>>>>> --
>>>>> Esta mensagem foi verificada pelo sistema de antivírus e
>>>>> acredita-se estar livre de perigo.
>>>>
>>>>
>>>> --
>>>> Esta mensagem foi verificada pelo sistema de antivírus e
>>>> acredita-se estar livre de perigo.
>>>
>>>
>>> --
>>> Esta mensagem foi verificada pelo sistema de antivírus e
>>> acredita-se estar livre de perigo.
>>
>>
>> --
>> Esta mensagem foi verificada pelo sistema de antivírus e
>> acredita-se estar livre de perigo.
>
>


[obm-l] Re: [obm-l] Re: [obm-l] Re: [obm-l] Re: [obm-l] Ajuda em teoria dos números

2020-10-22 Por tôpico Pedro José
Bom dia!
Recebi esse problema hoje: 615 + x^2 = 2^y., para x,y inteiros Não saberia
fazer, como não soube resolver esse, acima. Mas devido a solução do colega
Esdras, pensei:"já vi algo parecido".
Basta restringir y aos pares.
Se y é ímpar x^2=2 mod3, absurdo então y é par. Logo y=2a, com a inteiro.
(2^a + x) (2^a-x)= 615= 1*615=3*205=5*123=15*41 e como a soma dos fatores
necessita ser uma potência de 2, só serve para 123 e 5.
Logo 2^y=64 e y=6 e x= 59 ou x=-59.
Uma resolução levou a outra, não pelo talento nato, mas por aprendizado, o
que é válido.
Teve uma feita que estava tentando provar que o triângulo órtico, era o
triângulo de menor perímetro que poderia ser inscrito em um triângulo
acutângulo. Tentei por geometria analítica e só levando tinta. Tinha
desistido. Quando me deparei com um problema que não consegui resolver, mas
que tinha um caminho para a solução. Quando vi o rebatimento feito, pensei
está resolvido. O curioso, é que, quando desisti, pesquisei na internet e
não achei nada. Depois que consegui resolver, achei duas soluções, e
infelizmente e como esperado, a minha não era novidade, era clássica.
Obrigado, Esdras! Sem a sua solução, certamente, não teria resolvido essa
última questão.

Cordialmente,
PJMS

Em sex., 24 de jul. de 2020 às 12:19, Prof. Douglas Oliveira <
profdouglaso.del...@gmail.com> escreveu:

> Obrigado Claudio e Esdras, fatoração show
>
>
> Em sex., 24 de jul. de 2020 às 11:12, Esdras Muniz <
> esdrasmunizm...@gmail.com> escreveu:
>
>> Se for solução inteira positiva, acho que só tem 3 e 4. Vc supõe spdg x
>> maior ou igual a y, vê que y=1 não tem solução e x=y tb não. Daí, x>y>1.
>> Fatorando a expressão, fica: (xy-8-(x-y))(xy-8+(x-y))=15. Como
>> (xy-8-(x-y))>(xy-8+(x-y))>-2. Temos que ou (xy-8-(x-y))=1 e (xy-8+(x-y))=15,
>> o que não tem soluções inteiras positivas, ou (xy-8-(x-y))=3 e 
>> (xy-8+(x-y))=5,
>> cujas únicas soluções inteiras são x=4 e y=3.
>>
>> Em sex, 24 de jul de 2020 10:36, Claudio Buffara <
>> claudio.buff...@gmail.com> escreveu:
>>
>>> Pelo que entendi, a solução é a porção dessa curva algébrica situada no
>>> 1o quadrante.
>>> Dá pra fazer isso no Wolfram Alpha, com o comando plot (x*y-7)^2 - x^2 -
>>> y^2 = 0.
>>>
>>> []s,
>>> Claudio.
>>>
>>> On Fri, Jul 24, 2020 at 9:58 AM Prof. Douglas Oliveira <
>>> profdouglaso.del...@gmail.com> wrote:
>>>
 Preciso de ajuda para encontrar todas as soluções não negativas da
 equação
 (xy-7)^2=x^2+y^2.

 Desde já agradeço a ajuda
 Douglas Oliveira

 --
 Esta mensagem foi verificada pelo sistema de antivírus e
 acredita-se estar livre de perigo.
>>>
>>>
>>> --
>>> Esta mensagem foi verificada pelo sistema de antivírus e
>>> acredita-se estar livre de perigo.
>>
>>
>> --
>> Esta mensagem foi verificada pelo sistema de antivírus e
>> acredita-se estar livre de perigo.
>
>
> --
> Esta mensagem foi verificada pelo sistema de antivírus e
> acredita-se estar livre de perigo.


Re: [obm-l] Problema da IMO

2020-09-11 Por tôpico Pedro José
Boa noite!
Atrapalhou meu vinho e o filme que estava assistindo mas consegui. Não
gostei tanto, agora que consegui, é muito trabalhoso.

2= [3(y+1)(z+1)-1]/2yz
yz= 3(yz+2) (i)
z(y-3)= 3y +2 (ii)
y(z-3)=3z+2 (iii)
(i)*(ii) yz(z-3)(y-3)= 9yz+6(y+z)+4 e Voilá: (z-3)(y-3)=11.

Saudações,
PJMS





Em sáb., 12 de set. de 2020 às 00:35, Pedro José 
escreveu:

> Boa noite!
> Fui em uma linha parecida com a primeira solução, embora não visse
> necessidade de mudança de variáveis.
> Mas o b achei sempre por restrição.
> Esse "it implies" e aparece um número fatorado, não consegui captar,
> embora tenha gostado do recurso, já que é bem restritivo.
>
> Sudações,
> PJMS
>
>
> Em sáb., 12 de set. de 2020 às 00:08, Pedro José 
> escreveu:
>
>> Boa noite!
>> Grato, Ralph!
>>
>> Estou estudando a solução. Pelo menos, não me decepcionei. A resposta
>> estava correta,
>>
>> Saudações.
>> PJMS
>>
>> Em sex., 11 de set. de 2020 às 22:33, Ralph Costa Teixeira <
>> ralp...@gmail.com> escreveu:
>>
>>> Essa eh da IMO 1992. Tem uma solucao aqui:
>>> http://sms.math.nus.edu.sg/Simo/IMO_Problems/92.pdf
>>>
>>> On Fri, Sep 11, 2020 at 10:06 PM Pedro José  wrote:
>>>
>>>> Bom dia!
>>>>
>>>> Recebi de um filho de um amigo, um problema que já o fizera.
>>>> (a-1)(b-1)(c-1) | abc-1;  1>>>
>>>> Confesso que desta feita gastei mais tempo que da  primeira vez.
>>>> Curioso, da primeira ,eu pensei, dessa vez, eu tentei lembrar como eu
>>>> resolvera, aí nem lembrava, nem pensava. Apelei para a internet, mas não
>>>> encontrei nada. Mas no fim, recordei o que havia feito.
>>>> (1+1/(a-1))(1+1/(b-1))(1+1/(c-1)) = k, onde k é inteiro.
>>>> vê-se que k>1, e para um dado a k é máximo para b e c mínimos logo
>>>> b=a+1 e c=a+2
>>>> [a(a+1)(a+2)]/[(a-1)(a)(a+1)] > [a(a+1)(a+2)-1]/[(a-1)(a)(a+1)]>=2,
>>>> então (a+2)/(a-1)>2 ==> a <4, a=2 ou a=3.
>>>> O k é máximo para a=2, b=3 e c=4 ==> k <4, logo k=2 ou k=3.
>>>> S.p.g, se a é ímpar (a-1)(b-1)(c-1) é par; então b,c ímpares e k é
>>>> livre.
>>>> S.p.g se a é par abc-1 é ímpar; então b,c são pares e k ímpar.
>>>> a=2, temos  2b(b+1)/[(b-1)b] >3, não usei a restrição de paridade para
>>>> c para facilitar a simplificação. b<5 Logo a=2>>> k é ímpar k=3. Logo c= 8. (2,4,8) é uma solução.
>>>>
>>>> a=3 temos 3b(b+1)/[2(b-1)b] > 2; b<7 e 3>>> para a=3 e b=5. kmax <= (15*7-1)/(2*4*6) <=2;pois k é inteiro.
>>>> 1 k=2 e c= 15. (3,5,15) é a outra solução.
>>>>
>>>> Só agora me apercebi de que c=ab nas duas soluções. Então tentei uma
>>>> nova solução.
>>>> (a-1)(b-1)(c-1) | abc-1 e (a-1)(b-1)(c-1) | abc + c(1 - (a+b)) -ab+
>>>> (a+b) - 1 logo divide a diferença:
>>>> (a-1)(b-1)(c-1) | (a+b) (c-1) + ab -1 - (c-1) logo c-1 | ab-1, então
>>>> ab-1= w(c-1), para algum w inteiro e ab=w(c-1) +1 (i)
>>>> Como a=2 ou a=3
>>>> Se a=2. e w>=2
>>>> Temos por (i) 2b>= 2 (c-1) +1 c-1>=b, logo absurdo.
>>>> Se a=3
>>>> Temos por (i) 3b>= w(c-1)+1; w=3 ==>3b< 3 (c-1) +1 pois c>b
>>>> w=2 ==> 3b =2(c-1) +1 ==> c=(3b+1)/2
>>>> 2(b-1)(3b-1)/2 | 3b(3b+1)/2 -1 ==>  2(b-1)(3b-1)/ | 3b(3b+1) -2 ==>
>>>> 6b^2-8b-2 | 9b^2+3b-1 ==>  6b^2-8b-2 | 3b^2 +11b+ 4
>>>> ==> b <=5. Como b>a=3 ==> b=5 e c= 8, ferindo a paridade.
>>>> Logo ab-1=c-1 ==> ab=c ==> (a-1)(b-1)(c-1) | c^2-1 ==>  (a-1)(b-1) |
>>>> c+1 (a-1)(b-1) |ab+1==> (a-1)(b-1)!a+b
>>>> a=2 ==> 2b-2= 2+b. b=4 e c=ab=8 (2,4,8)
>>>> a=3 ==> 2(b-1) | 3+b ==> 2(b-1) = 3+. b=5 e c=ab=15. (3,5,15),
>>>> Forcei um pouco a barra para mostrar que c=ab.
>>>> Alguém teria uma outra solução, ou um endereço onde se tem as questões
>>>> da IMO e suas resoluções?
>>>>
>>>> Grato!
>>>> Saudações,
>>>> PJMS
>>>>
>>>>
>>>>
>>>> --
>>>> Esta mensagem foi verificada pelo sistema de antivírus e
>>>> acredita-se estar livre de perigo.
>>>
>>>
>>> --
>>> Esta mensagem foi verificada pelo sistema de antivírus e
>>> acredita-se estar livre de perigo.
>>
>>

-- 
Esta mensagem foi verificada pelo sistema de antiv�rus e
 acredita-se estar livre de perigo.



Re: [obm-l] Problema da IMO

2020-09-11 Por tôpico Pedro José
Boa noite!
Fui em uma linha parecida com a primeira solução, embora não visse
necessidade de mudança de variáveis.
Mas o b achei sempre por restrição.
Esse "it implies" e aparece um número fatorado, não consegui captar, embora
tenha gostado do recurso, já que é bem restritivo.

Sudações,
PJMS


Em sáb., 12 de set. de 2020 às 00:08, Pedro José 
escreveu:

> Boa noite!
> Grato, Ralph!
>
> Estou estudando a solução. Pelo menos, não me decepcionei. A resposta
> estava correta,
>
> Saudações.
> PJMS
>
> Em sex., 11 de set. de 2020 às 22:33, Ralph Costa Teixeira <
> ralp...@gmail.com> escreveu:
>
>> Essa eh da IMO 1992. Tem uma solucao aqui:
>> http://sms.math.nus.edu.sg/Simo/IMO_Problems/92.pdf
>>
>> On Fri, Sep 11, 2020 at 10:06 PM Pedro José  wrote:
>>
>>> Bom dia!
>>>
>>> Recebi de um filho de um amigo, um problema que já o fizera.
>>> (a-1)(b-1)(c-1) | abc-1;  1>>
>>> Confesso que desta feita gastei mais tempo que da  primeira vez.
>>> Curioso, da primeira ,eu pensei, dessa vez, eu tentei lembrar como eu
>>> resolvera, aí nem lembrava, nem pensava. Apelei para a internet, mas não
>>> encontrei nada. Mas no fim, recordei o que havia feito.
>>> (1+1/(a-1))(1+1/(b-1))(1+1/(c-1)) = k, onde k é inteiro.
>>> vê-se que k>1, e para um dado a k é máximo para b e c mínimos logo b=a+1
>>> e c=a+2
>>> [a(a+1)(a+2)]/[(a-1)(a)(a+1)] > [a(a+1)(a+2)-1]/[(a-1)(a)(a+1)]>=2,
>>> então (a+2)/(a-1)>2 ==> a <4, a=2 ou a=3.
>>> O k é máximo para a=2, b=3 e c=4 ==> k <4, logo k=2 ou k=3.
>>> S.p.g, se a é ímpar (a-1)(b-1)(c-1) é par; então b,c ímpares e k é livre.
>>> S.p.g se a é par abc-1 é ímpar; então b,c são pares e k ímpar.
>>> a=2, temos  2b(b+1)/[(b-1)b] >3, não usei a restrição de paridade para c
>>> para facilitar a simplificação. b<5 Logo a=2>> é ímpar k=3. Logo c= 8. (2,4,8) é uma solução.
>>>
>>> a=3 temos 3b(b+1)/[2(b-1)b] > 2; b<7 e 3>> para a=3 e b=5. kmax <= (15*7-1)/(2*4*6) <=2;pois k é inteiro.
>>> 1 k=2 e c= 15. (3,5,15) é a outra solução.
>>>
>>> Só agora me apercebi de que c=ab nas duas soluções. Então tentei uma
>>> nova solução.
>>> (a-1)(b-1)(c-1) | abc-1 e (a-1)(b-1)(c-1) | abc + c(1 - (a+b)) -ab+
>>> (a+b) - 1 logo divide a diferença:
>>> (a-1)(b-1)(c-1) | (a+b) (c-1) + ab -1 - (c-1) logo c-1 | ab-1, então
>>> ab-1= w(c-1), para algum w inteiro e ab=w(c-1) +1 (i)
>>> Como a=2 ou a=3
>>> Se a=2. e w>=2
>>> Temos por (i) 2b>= 2 (c-1) +1 c-1>=b, logo absurdo.
>>> Se a=3
>>> Temos por (i) 3b>= w(c-1)+1; w=3 ==>3b< 3 (c-1) +1 pois c>b
>>> w=2 ==> 3b =2(c-1) +1 ==> c=(3b+1)/2
>>> 2(b-1)(3b-1)/2 | 3b(3b+1)/2 -1 ==>  2(b-1)(3b-1)/ | 3b(3b+1) -2 ==>
>>> 6b^2-8b-2 | 9b^2+3b-1 ==>  6b^2-8b-2 | 3b^2 +11b+ 4
>>> ==> b <=5. Como b>a=3 ==> b=5 e c= 8, ferindo a paridade.
>>> Logo ab-1=c-1 ==> ab=c ==> (a-1)(b-1)(c-1) | c^2-1 ==>  (a-1)(b-1) |
>>> c+1 (a-1)(b-1) |ab+1==> (a-1)(b-1)!a+b
>>> a=2 ==> 2b-2= 2+b. b=4 e c=ab=8 (2,4,8)
>>> a=3 ==> 2(b-1) | 3+b ==> 2(b-1) = 3+. b=5 e c=ab=15. (3,5,15),
>>> Forcei um pouco a barra para mostrar que c=ab.
>>> Alguém teria uma outra solução, ou um endereço onde se tem as questões
>>> da IMO e suas resoluções?
>>>
>>> Grato!
>>> Saudações,
>>> PJMS
>>>
>>>
>>>
>>> --
>>> Esta mensagem foi verificada pelo sistema de antivírus e
>>> acredita-se estar livre de perigo.
>>
>>
>> --
>> Esta mensagem foi verificada pelo sistema de antivírus e
>> acredita-se estar livre de perigo.
>
>

-- 
Esta mensagem foi verificada pelo sistema de antiv�rus e
 acredita-se estar livre de perigo.



Re: [obm-l] Problema da IMO

2020-09-11 Por tôpico Pedro José
Boa noite!
Grato, Ralph!

Estou estudando a solução. Pelo menos, não me decepcionei. A resposta
estava correta,

Saudações.
PJMS

Em sex., 11 de set. de 2020 às 22:33, Ralph Costa Teixeira <
ralp...@gmail.com> escreveu:

> Essa eh da IMO 1992. Tem uma solucao aqui:
> http://sms.math.nus.edu.sg/Simo/IMO_Problems/92.pdf
>
> On Fri, Sep 11, 2020 at 10:06 PM Pedro José  wrote:
>
>> Bom dia!
>>
>> Recebi de um filho de um amigo, um problema que já o fizera.
>> (a-1)(b-1)(c-1) | abc-1;  1>
>> Confesso que desta feita gastei mais tempo que da  primeira vez. Curioso,
>> da primeira ,eu pensei, dessa vez, eu tentei lembrar como eu resolvera, aí
>> nem lembrava, nem pensava. Apelei para a internet, mas não encontrei nada.
>> Mas no fim, recordei o que havia feito.
>> (1+1/(a-1))(1+1/(b-1))(1+1/(c-1)) = k, onde k é inteiro.
>> vê-se que k>1, e para um dado a k é máximo para b e c mínimos logo b=a+1
>> e c=a+2
>> [a(a+1)(a+2)]/[(a-1)(a)(a+1)] > [a(a+1)(a+2)-1]/[(a-1)(a)(a+1)]>=2, então
>> (a+2)/(a-1)>2 ==> a <4, a=2 ou a=3.
>> O k é máximo para a=2, b=3 e c=4 ==> k <4, logo k=2 ou k=3.
>> S.p.g, se a é ímpar (a-1)(b-1)(c-1) é par; então b,c ímpares e k é livre.
>> S.p.g se a é par abc-1 é ímpar; então b,c são pares e k ímpar.
>> a=2, temos  2b(b+1)/[(b-1)b] >3, não usei a restrição de paridade para c
>> para facilitar a simplificação. b<5 Logo a=2> é ímpar k=3. Logo c= 8. (2,4,8) é uma solução.
>>
>> a=3 temos 3b(b+1)/[2(b-1)b] > 2; b<7 e 3> para a=3 e b=5. kmax <= (15*7-1)/(2*4*6) <=2;pois k é inteiro.
>> 1 k=2 e c= 15. (3,5,15) é a outra solução.
>>
>> Só agora me apercebi de que c=ab nas duas soluções. Então tentei uma nova
>> solução.
>> (a-1)(b-1)(c-1) | abc-1 e (a-1)(b-1)(c-1) | abc + c(1 - (a+b)) -ab+ (a+b)
>> - 1 logo divide a diferença:
>> (a-1)(b-1)(c-1) | (a+b) (c-1) + ab -1 - (c-1) logo c-1 | ab-1, então
>> ab-1= w(c-1), para algum w inteiro e ab=w(c-1) +1 (i)
>> Como a=2 ou a=3
>> Se a=2. e w>=2
>> Temos por (i) 2b>= 2 (c-1) +1 c-1>=b, logo absurdo.
>> Se a=3
>> Temos por (i) 3b>= w(c-1)+1; w=3 ==>3b< 3 (c-1) +1 pois c>b
>> w=2 ==> 3b =2(c-1) +1 ==> c=(3b+1)/2
>> 2(b-1)(3b-1)/2 | 3b(3b+1)/2 -1 ==>  2(b-1)(3b-1)/ | 3b(3b+1) -2 ==>
>> 6b^2-8b-2 | 9b^2+3b-1 ==>  6b^2-8b-2 | 3b^2 +11b+ 4
>> ==> b <=5. Como b>a=3 ==> b=5 e c= 8, ferindo a paridade.
>> Logo ab-1=c-1 ==> ab=c ==> (a-1)(b-1)(c-1) | c^2-1 ==>  (a-1)(b-1) |  c+1
>> (a-1)(b-1) |ab+1==> (a-1)(b-1)!a+b
>> a=2 ==> 2b-2= 2+b. b=4 e c=ab=8 (2,4,8)
>> a=3 ==> 2(b-1) | 3+b ==> 2(b-1) = 3+. b=5 e c=ab=15. (3,5,15),
>> Forcei um pouco a barra para mostrar que c=ab.
>> Alguém teria uma outra solução, ou um endereço onde se tem as questões da
>> IMO e suas resoluções?
>>
>> Grato!
>> Saudações,
>> PJMS
>>
>>
>>
>> --
>> Esta mensagem foi verificada pelo sistema de antivírus e
>> acredita-se estar livre de perigo.
>
>
> --
> Esta mensagem foi verificada pelo sistema de antivírus e
> acredita-se estar livre de perigo.

-- 
Esta mensagem foi verificada pelo sistema de antiv�rus e
 acredita-se estar livre de perigo.



[obm-l] Problema da IMO

2020-09-11 Por tôpico Pedro José
Bom dia!

Recebi de um filho de um amigo, um problema que já o fizera.
(a-1)(b-1)(c-1) | abc-1;  11, e para um dado a k é máximo para b e c mínimos logo b=a+1 e
c=a+2
[a(a+1)(a+2)]/[(a-1)(a)(a+1)] > [a(a+1)(a+2)-1]/[(a-1)(a)(a+1)]>=2, então
(a+2)/(a-1)>2 ==> a <4, a=2 ou a=3.
O k é máximo para a=2, b=3 e c=4 ==> k <4, logo k=2 ou k=3.
S.p.g, se a é ímpar (a-1)(b-1)(c-1) é par; então b,c ímpares e k é livre.
S.p.g se a é par abc-1 é ímpar; então b,c são pares e k ímpar.
a=2, temos  2b(b+1)/[(b-1)b] >3, não usei a restrição de paridade para c
para facilitar a simplificação. b<5 Logo a=2 2; b<7 e 3 k=2 e c= 15. (3,5,15) é a outra solução.

Só agora me apercebi de que c=ab nas duas soluções. Então tentei uma nova
solução.
(a-1)(b-1)(c-1) | abc-1 e (a-1)(b-1)(c-1) | abc + c(1 - (a+b)) -ab+ (a+b) -
1 logo divide a diferença:
(a-1)(b-1)(c-1) | (a+b) (c-1) + ab -1 - (c-1) logo c-1 | ab-1, então ab-1=
w(c-1), para algum w inteiro e ab=w(c-1) +1 (i)
Como a=2 ou a=3
Se a=2. e w>=2
Temos por (i) 2b>= 2 (c-1) +1 c-1>=b, logo absurdo.
Se a=3
Temos por (i) 3b>= w(c-1)+1; w=3 ==>3b< 3 (c-1) +1 pois c>b
w=2 ==> 3b =2(c-1) +1 ==> c=(3b+1)/2
2(b-1)(3b-1)/2 | 3b(3b+1)/2 -1 ==>  2(b-1)(3b-1)/ | 3b(3b+1) -2 ==>
6b^2-8b-2 | 9b^2+3b-1 ==>  6b^2-8b-2 | 3b^2 +11b+ 4
==> b <=5. Como b>a=3 ==> b=5 e c= 8, ferindo a paridade.
Logo ab-1=c-1 ==> ab=c ==> (a-1)(b-1)(c-1) | c^2-1 ==>  (a-1)(b-1) |  c+1
(a-1)(b-1) |ab+1==> (a-1)(b-1)!a+b
a=2 ==> 2b-2= 2+b. b=4 e c=ab=8 (2,4,8)
a=3 ==> 2(b-1) | 3+b ==> 2(b-1) = 3+. b=5 e c=ab=15. (3,5,15),
Forcei um pouco a barra para mostrar que c=ab.
Alguém teria uma outra solução, ou um endereço onde se tem as questões da
IMO e suas resoluções?

Grato!
Saudações,
PJMS

-- 
Esta mensagem foi verificada pelo sistema de antiv�rus e
 acredita-se estar livre de perigo.



[obm-l] Re: [obm-l] É um número?

2020-08-27 Por tôpico Pedro Porta
Boa noite, caso seja perante as duas condições não, se trata de um valor
numérico irrepresentável.

Em qui, 27 de ago de 2020 17:30, marcone augusto araújo borges <
marconeborge...@hotmail.com> escreveu:

> Faz sentido a^x, se a< 0 e x é irracional positivo?
> --
> Esta mensagem foi verificada pelo sistema de antivírus e
> acredita-se estar livre de perigo.
>

-- 
Esta mensagem foi verificada pelo sistema de antiv�rus e
 acredita-se estar livre de perigo.



[obm-l] Re: [obm-l] Re: [obm-l] Re: [obm-l] Re: [obm-l] Re: [obm-l] Geometria plana com desigualdade de médias?

2020-08-26 Por tôpico Pedro José
Boa noite!
Anderson,
achei legal a sua visão. Mas não consegui evoluir com nada.
Todavia, fiquei com uma dúvida. Como x+y é um dos ângulos do triângulo
temos a restrição 0 escreveu:

> Em qui., 20 de ago. de 2020 às 22:03, Anderson Torres
>  escreveu:
> >
> > Em ter., 18 de ago. de 2020 às 19:51, Pedro José 
> escreveu:
> > >
> > > Boa noite!
> > > Cláudio,
> > > não consegui nada geométrico.
> > > O máximo que atingi foi:
> > > a/ha + b/hb + c/hc= [cotg(A1) +cotg (A2)]  + [cotg(B1) +cotg (B2)] +
> co[tg(C1) +cotg (C2)] com A1 + A2 = A; B1 + B2 + B e C1 + C2 = C.
> > > Para ser mínimo cada termo entre colchetes deve ser mínimo, o que
> ocorre quando A1 = A2; B1 = B2 e C1 = C2. Logo P seria o encontro das
> bissetrizes e logo I.
> > > Onde: A1= PAB e A2=PAC; B1=PBA e B2=PBC; C1=PCA e C2=PCB.
> >
> > Acho que daqui poderia sair uma interpretação mais escamoteada.
> > Afinal, trigonometria é uma espécie de "ponto de contato" entre a
> > geometria analítica e a sintética, entre a nuvem de desenhos e a de
> > números.
> >
> > Acredito que a solução aqui seria arranjar uma interpretação
> > geométrica desses colchetes de co-tangentes. Acredito que possamos
> > apelar para Ptolomeu em algum momento ou para um macete de
> > semelhanças, pois as projeções de um ponto sobre duas retas criam um
> > quadrilátero cíclico.
>
> Acrescentando mais coisas: se queremos minimizar cot(x) +cot(y) com
> x+y fixo, isto é equivalente a minimizar tan(90-x)+tan(90-y) com
> 90-x+90-y fixo. Ou como maximizar tan(x) + tan(y) com x+y fixo.
>
> Geometricamente, tangente é cateto oposto dividido por cateto
> adjacente. Logo uma soma de tangentes com catetos adjacentes iguais
> equivale a uma soma de catetos opostos! Assim sendo, nosso problema
> pode ser pensado da seguinte forma:
>
> Dados um ponto A e uma reta d fixos, temos que construir duas retas x
> e y, com ângulo 'alfa' entre elas, ambas passando por A e tais que a
> distância entre os pontos X e Y, que elas geram ao intersectar d, seja
> mínima.
>
> Daí fica fácil argumentar que a altura por A também tem que ser a
> bissetriz por A.
>
> No fundo do fundo é uma forma de geometrizar a solução trigonométrica.
> A trigonometria se torna apenas um atalho.
>
> Vou formalizar isso mais tarde, com desenhos e tudo.
>
>
>
> >
> > Isso até me lembra o famoso artigo do Shine sobre geometria cearense
> > VS geometria paulista:
> > https://cyshine.webs.com/geometria-2005.pdf
> >
> >
> > >
> > > Saudações,
> > > PJMS
> > >
> > > Em ter., 18 de ago. de 2020 às 11:34, Claudio Buffara <
> claudio.buff...@gmail.com> escreveu:
> > >>
> > >> Será que tem uma demonstração mais geométrica e menos algébrica
> disso? E que torne o resultado mais intuitivo?
> > >> É razoável que o ponto P não esteja muito próximo de qualquer dos
> lados, pois neste caso, se P se aproximasse do lado a, por exemplo, a/h_a
> cresceria e a expressão se afastaria do valor mínimo.
> > >> Mas, com lados não necessariamente congruentes, não é óbvio, a
> priori, que P deva ser equidistante dos três.
> > >> De fato, seria razoável esperar que P estivesse mais próximo do maior
> lado e conjecturar, por exemplo, que o P que minimiza a expressão é tal que
> a/h_a = b/h_b = c/h_c.
> > >> O fato de P ser o incentro não me parece a conjectura mais evidente
> neste caso.
> > >>
> > >>
> > >> On Sun, Aug 16, 2020 at 10:11 AM Matheus Secco <
> matheusse...@gmail.com> wrote:
> > >>>
> > >>> Olá, Vanderlei.
> > >>> Por Cauchy-Schwarz, temos
> > >>>
> > >>> (a/ha + b/hb + c/hc) * (a*ha + b*hb + c*hc) >= (a+b+c)^2.  (#)
> > >>>
> > >>> Como (a*ha + b*hb + c*hc) = 2S, onde S é a área de ABC, segue que a
> expressão a/ha + b/hb + c/hc é pelo menos 2p^2/S, onde p é o semi-perimetro.
> > >>>
> > >>> Por outro lado, a igualdade em (#) ocorre se, e somente se, ha = hb
> = hc, ou seja, quando P é o incentro do triângulo
> > >>>
> > >>> Abraços,
> > >>> Matheus
> > >>>
> > >>> Em dom, 16 de ago de 2020 08:59, Professor Vanderlei Nemitz <
> vanderma...@gmail.com> escreveu:
> > >>>>
> > >>>> Bom dia!
> > >>>>
> > >>>> Tentei utilizar alguma desigualdade de médias aqui, mas não tive
> êxito. Alguém ajuda?
> > >>>> Muito agradecido!
> > >>>>
>

[obm-l] Re: [obm-l] Re: [obm-l] Re: [obm-l] Geometria plana com desigualdade de médias?

2020-08-18 Por tôpico Pedro José
Boa noite!
Cláudio,
não consegui nada geométrico.
O máximo que atingi foi:
a/ha + b/hb + c/hc= [cotg(A1) +cotg (A2)]  + [cotg(B1) +cotg (B2)] +
co[tg(C1) +cotg (C2)] com A1 + A2 = A; B1 + B2 + B e C1 + C2 = C.
Para ser mínimo cada termo entre colchetes deve ser mínimo, o que ocorre
quando A1 = A2; B1 = B2 e C1 = C2. Logo P seria o encontro das bissetrizes
e logo I.
Onde: A1= PAB e A2=PAC; B1=PBA e B2=PBC; C1=PCA e C2=PCB.

Saudações,
PJMS

Em ter., 18 de ago. de 2020 às 11:34, Claudio Buffara <
claudio.buff...@gmail.com> escreveu:

> Será que tem uma demonstração mais geométrica e menos algébrica disso? E
> que torne o resultado mais intuitivo?
> É razoável que o ponto P não esteja muito próximo de qualquer dos lados,
> pois neste caso, se P se aproximasse do lado a, por exemplo,
> a/h_a cresceria e a expressão se afastaria do valor mínimo.
> Mas, com lados não necessariamente congruentes, não é óbvio, a priori, que
> P deva ser equidistante dos três.
> De fato, seria razoável esperar que P estivesse mais próximo do maior lado
> e conjecturar, por exemplo, que o P que minimiza a expressão é tal que
> a/h_a = b/h_b = c/h_c.
> O fato de P ser o incentro não me parece a conjectura mais evidente neste
> caso.
>
>
> On Sun, Aug 16, 2020 at 10:11 AM Matheus Secco 
> wrote:
>
>> Olá, Vanderlei.
>> Por Cauchy-Schwarz, temos
>>
>> (a/ha + b/hb + c/hc) * (a*ha + b*hb + c*hc) >= (a+b+c)^2.  (#)
>>
>> Como (a*ha + b*hb + c*hc) = 2S, onde S é a área de ABC, segue que a
>> expressão a/ha + b/hb + c/hc é pelo menos 2p^2/S, onde p é o
>> semi-perimetro.
>>
>> Por outro lado, a igualdade em (#) ocorre se, e somente se, ha = hb = hc,
>> ou seja, quando P é o incentro do triângulo
>>
>> Abraços,
>> Matheus
>>
>> Em dom, 16 de ago de 2020 08:59, Professor Vanderlei Nemitz <
>> vanderma...@gmail.com> escreveu:
>>
>>> Bom dia!
>>>
>>> Tentei utilizar alguma desigualdade de médias aqui, mas não tive êxito.
>>> Alguém ajuda?
>>> Muito agradecido!
>>>
>>> Seja P um ponto no interior de um triângulo e sejam ha, hb e hc as
>>> distâncias de P aos lados a, b e c, respectivamente. Mostre que o valor
>>> mínimo de (a/ha) + (b/hb) + (c/hc) ocorre quando P é o incentivo do
>>> triângulo ABC.
>>>
>>> --
>>> Esta mensagem foi verificada pelo sistema de antivírus e
>>> acredita-se estar livre de perigo.
>>
>>
>> --
>> Esta mensagem foi verificada pelo sistema de antivírus e
>> acredita-se estar livre de perigo.
>
>
> --
> Esta mensagem foi verificada pelo sistema de antivírus e
> acredita-se estar livre de perigo.

-- 
Esta mensagem foi verificada pelo sistema de antiv�rus e
 acredita-se estar livre de perigo.



[obm-l] Re: [obm-l] Re: [obm-l] polinômio minimal

2020-08-09 Por tôpico João Pedro de Abreu Marciano
Verdade...
Seja p = x^8 - 12 x^6 + 32 x^4 - 72 x^2 + 4 um polinômio minimal de α,
então não pode haver polinômio de grau menor que 8 com α sendo raiz.
Suponha que p não é irredutível. Logo, existem g,h tais que  p = g*h, com
0
escreveu:

> Sauda,c~oes, oi João Pedro,
>
> Obrigado por responder.
>
> Tinha feito isso. Deu
>
> 
>
> expand  (x + 1)^8 - 12 (x + 1)^6 + 32 (x + 1)^4 - 72 (x + 1)^2 + 4
>
> x^8 + 8 x^7 + 16 x^6 - 16 x^5 - 78 x^4 - 56 x^3 - 32 x^2 - 80 x - 47
>
> E o Critério de Eisenstein não se aplica.
>
> E para x=x+2, deu
>
> x^8 + 16 x^7 + 100 x^6 + 304 x^5 + 432 x^4 + 128 x^3 - 392 x^2 - 544 x -
> 284
>
> Quase. Falha no a_0=284.
>
> Luís
>
>
>

-- 
Esta mensagem foi verificada pelo sistema de antiv�rus e
 acredita-se estar livre de perigo.



[obm-l] Re: [obm-l] polinômio minimal

2020-08-08 Por tôpico João Pedro de Abreu Marciano
Boa noite!

Tente aplicar o Critério de Eisenstein com p=3 e substituindo x por x+1.

Att.
João Pedro.

Em sáb., 8 de ago. de 2020 às 17:14, 
escreveu:

> Sauda,c~oes,
>
> O polinômio 
> é o polinômio minimal de α = sqrt(2) + sqrt(1+sqrt(3)).
>
> Como provar que ele é irredutível em Q[x] ?
>
> Luís
>
> --
> Esta mensagem foi verificada pelo sistema de antivírus e
> acredita-se estar livre de perigo.

-- 
Esta mensagem foi verificada pelo sistema de antiv�rus e
 acredita-se estar livre de perigo.



[obm-l] Problema da IMO

2020-07-23 Por tôpico Pedro José
 Encontre todos os (k,n), k,n pertencentes à Z+, tal que k!=
(2^n-1)*(2^n-2)*(2^n-4)*...(2^n-2^(n-2))*(2^n-2(n-1))

Gostaria de saber se está correto?

Como os dois termos iniciais são consecutivos, é intuitivo que haja
baixíssima probabilidade de termos respostas que não sejam as triviais, com
um ou dois fatores.
(1,1) ==> 1! = 2-1 correto! e (3,2) --> 3!= 3*2, correto!
Para n= 3 temos k!= 7*6*4 com um fator 7 e não temos um 5, não atende.
Para n=4 temos k!=15*14*12*8=20160, mas 7! <2 0.060 < 8!;não atende.

G(n) = (2^n-1)*(2^n-2)*(2^n-4)*...(2^n-2^(n-2))*(2^n-2(n-1)) ==> G(n+1)=
(2^(n+1) -1)**G(n)2^n

Vamos mostrar por absurdo que k! <>
(2^n-1)*(2^n-2)*(2^n-4)*...(2^n-2^(n-2))*(2^n-2(n-1))para qualquer n>=5.
Seja , por hipótese, k! =
(2^n-1)*(2^n-2)*(2^n-4)*...(2^n-2^(n-2))*(2^n-2(n-1))e n>=5.
Fica claro que 2^i || (2^n-2î) com n>i e n, i inteiros.
Então obtém-se uma P.A. para o expoente da fatoração de cada termo do
produto, nos levando a: k!=2^((n-1)*n/2)*m, com m ímpar.Ou seja, a
fatoração de k! tem como o expoente de 2, a= (n-1)*n/2
Todavia, por contagem é muito simples chegar-se a: a=
[k/2]+[k/4]+[k/8]+[k/16]+...; Note que embora haja uma infinidade de
parcela, haverá um j, tal que 2^j>k e a partir desse termo todas as
parcelas irão zerar. Onde [x] representa, parte inteira de x.
a= [k/2]+[k/4]+[k/8]+[k/16]+...= [k/2]+[k/4]+[k/8]+[k/16]+ ...[k/2^(j-1] <
[k/2]+[k/4] + ...+[k/2^(j-1)] + k/2^j + k/2^(j+1)+ k/2^(j+2+< k/2 + k/4
+ k/8 + k/16+...= k ==>
==> k>a ==> k>= (a+1), já que k e a são inteiros. então k! >=(a+1)!
Vamos por indução.
Para n=5 a=10 e k!>=11! > 31*30*28*24*16=k!, por hipótese; k! >k!, absurdo.

Se k!>=(a+1)! > (2^n-1)*(2^n-2)*(2^n-4)*...(2^n-2^(n-2))*(2^n-2(n-1)) para
n>=5
temos F=((n-1)*n/2+1)*(n-1)*n/2*((n-1)*n/2-1)*...2*1>
(2^n-1)*(2^n-2)*(2^n-4)*...(2^n-2^(n-2))*(2^n-2(n-1)=P

Para n+1
k! >= (a+1)! >
(n*(n+1)/2+1)*n*(n+1)/2*(n*(n+1)/2-1)**((n-1)*n/2+2)*F>(2^(n+1)-1)*2*(2^n-1)*2(2^n-2)*...*2*(2^n-2(n-1)=
(2^(n+1)-1)*2^n*P
Mas como F> P se mostramos que
(n*(n+1)/2+1)*n*(n+1)/2*(n*(n+1)/2-1)**((n-1)*n/2+2)>2^n*(2^(n+1)*2^n,
está provado.
O lado esquerdo é composto de n fatores todos maiores que o último que é
maior ou igual 12, para n=5.
O lado esquerdo é menor que 2^(2n+1)
Como 12^n= 3^n*2^2^n > 2* 2^2n > 2^n*(2^(n+1)*2^n,==>
(n*(n+1)/2+1)*n*(n+1)/2*(n*(n+1)/2-1)**((n-1)*n/2+2)>2^n*(2^(n+1)*2^.
então k!>k!, absurdo. Não existem soluções para k>=5. E como só existiam
duas para k<5, temos S= {(1,1);(3,2)}

Saudações,
PJMS

-- 
Esta mensagem foi verificada pelo sistema de antiv�rus e
 acredita-se estar livre de perigo.



Re: [obm-l] Normas

2020-06-15 Por tôpico João Pedro de Abreu Marciano
Dado M>1. Definimos f(x) = 0 se 1/M0 tal que | f |_infinito <= B*|
f |_1 para todo f. Ou seja, as normas não são equivalentes.

Espero ter ajudado,
João Pedro Marciano.

Em seg., 15 de jun. de 2020 às 22:46, Pedro Júnior <
pedromatematic...@gmail.com> escreveu:

> [image: image.png]
> Alguém pode me ajudar nesse problema?
>
> --
>
> Pedro Jerônimo S. de O. Júnior
>
> Professor de Matemática
>
> João Pessoa – PB
>
> --
> Esta mensagem foi verificada pelo sistema de antivírus e
> acredita-se estar livre de perigo.

-- 
Esta mensagem foi verificada pelo sistema de antiv�rus e
 acredita-se estar livre de perigo.



[obm-l] Normas

2020-06-15 Por tôpico Pedro Júnior
[image: image.png]
Alguém pode me ajudar nesse problema?

-- 

Pedro Jerônimo S. de O. Júnior

Professor de Matemática

João Pessoa – PB

-- 
Esta mensagem foi verificada pelo sistema de antiv�rus e
 acredita-se estar livre de perigo.



[obm-l] Re: [obm-l] Decrescimento de Funções Exponenciais

2020-05-12 Por tôpico Pedro Angelo
Sobre o item 5, o que acontece se h(x)=x^(-1) e g(x)=x^(-1.1) ?

Le mar. 12 mai 2020 à 09:52, Luiz Antonio Rodrigues
 a écrit :
>
> Olá, pessoal!
>
> Bom dia!
>
> Tudo bem?
>
> Estou tentando resolver um problema há uns 10 dias.
>
> Já tentei de tudo e estou com dúvidas.
>
> O problema é o seguinte:
>
>
> São dadas duas funções: h(x) e g(x).
>
> A função g(x) tende a zero mais rápido do que h(x), quando x tende a infinito.
>
> O problema pede que as seguintes funções sejam comparadas com h(x):
>
>
> (h(x))^2
>
> (g(x))^2
>
> f(x)*g(x)
>
> sqrt(h(x))
>
> sqrt(g(x))
>
>
> A pergunta é: quais dessas funções decrescem mais rápido do que h(x), quando 
> x tende a infinito?
>
> Eu usei, entre outras, as seguintes funções:
>
>
> 1/ln(x)
>
> 1/x
>
> 1/x^5
>
> 1/e^x
>
>
> Utilizei a regra de L’Hospital e descobri que a única função que não decresce 
> mais rápido do que h(x) é a (4).
>
> Também utilizei softwares gráficos e confirmei o meu resultado.
>
> Só sei que a resposta não está correta, mas ainda não sei qual seria a 
> solução.
>
> Não consigo entender o motivo...
>
> Será que preciso achar um contra-exemplo?
>
> Alguém pode me ajudar?
>
> Muito obrigado!
>
> Abraços!
>
> Luiz
>
>
> --
> Esta mensagem foi verificada pelo sistema de antivírus e
> acredita-se estar livre de perigo.

-- 
Esta mensagem foi verificada pelo sistema de antiv�rus e
 acredita-se estar livre de perigo.


=
Instru��es para entrar na lista, sair da lista e usar a lista em
http://www.mat.puc-rio.br/~obmlistas/obm-l.html
=


Re: [obm-l] RE: Dois problemas

2020-04-27 Por tôpico Pedro José
Mandei espuriamente. Corrigi o que faltava.

Seja b=7
n é formado por 17C7 e R(n) por 7C71, logo 4*c+3 >=30 e 4c+3=7 mod 10,
portanto não há c que atenda.
Seja b=9
n é formado por 19C7 e R(n) por 7C91, logo 4c+3=9 mod 10 e
4*9+int((4c+3)/10)=c mod10
Só atende a primeira 4 ou 9, desses só atende a segunda 9.

Logo n= 1997 e 4n +3= 7991, resposta única. Havia mandado adiantado.


Em seg, 27 de abr de 2020 21:19, Pedro José  escreveu:

> Boa noite!
> 1) a_n = [n(n+1)]/2 mod 10
> Fácilmente se vê que 20 é múltiplo do período minímo pois:
> a_19= 0 e a_20=0 logo a_21= a_20+21mod10 =0+1=1=a1.
> a_22=a_1+22 mod10 = 3=a_3 É assim sucessivamente.
> Então o período é um divisor de 20
> p<>1, pois, a_1<>a_2
> p<>2, pois, a_1<>a_3
> p<>4, pois a_1<>a_5
> p<>5, pois a_1<> a_6
> p<>10 pois a_1<>a_11
> logo p = 20.
> Não vem outra forma se calcular a_1 até a_20
>
> a_1=1 e S_1=1
> a_2=3  e S_2=4
> a_3=6 e S_3=10
> a_4=0 e S_4=10
> a_5=5 e S_5=15
> a_6=1 e S_6= 16
> a_7=8 e S_7=24
> a_8=6 e S_8=30
> a_9=5 e S_9=35
> a_10=5 e S_10=40
> a_11=6 e S_11=46
> a_12=8 e S_12=54
> a_13=1 e S_13=55
> a_14=5 e S_14=60
> a_15=0 e S_15=60
> a_16=6 e S_16= 66
> a_17=3 e S_17=69
> a_18=1 e S_18=70
> a_19=0 e S_19=70
> a_20=0 e S_20=70
> É fácil ver que a cada 20 números a mais a soma dará 70 de novo.,  1992 =
> 20 x 99 +12; logo S_1992 =99 S_×0+S_12=6984.
>
> 2) seja n formado por ABCD, vamos representar a ideia número pela letra
> minúscula a, b, c, d e U={a, b, c, d} max(U)=9 e min(U)=0.
> a<=2 e a ímpar pois R(n)=DCBA é ímpar.
> Então a=1
>  4n+3= 4x(a*10^3+b*10^2+c*10+d)+3=
> =4d+3 mod10 4d+3=a mod10 ==> d=2 ou d=7, mas d>=4 pois 4n+3= d*10^3 +
> c*10^2 + b*10 + a > 4 *10^3. Logo d=7
> Logo b=7 ou b=9
> pois 4*b +3>30 É b é ímpar pois
> DCBA é côngruo 3 mod4
> Seja b=7
> n é formado por 17C7 e R(n) por 7C71, logo 4*c+3 >=30 e 4c+3=7 mod 10,
> portanto não há c que atenda.
> Seja b=9
> n é formado por 19C7 e R(n) por 7C91, logo 4c+3=9 mod 10 e
> 4*9+int((4c+3)/10)=c mod10
> Só atende a primeira 4 ou 9, desses só atende a segunda 9.
>
Logo n= 1997 e 4n +3= 7991, resposta única. Havia mandado adiantado.


>
>
>
>
> Em dom, 26 de abr de 2020 22:56, Julio Mohnsam <
> prof.juliomat...@hotmail.com> escreveu:
>
>> se n=2019
>>
>> --
>> *De:* owner-ob...@mat.puc-rio.br  em nome de
>> Rogério Possi Júnior 
>> *Enviado:* domingo, 26 de abril de 2020 18:21
>> *Para:* Lista de Olímpiada OBM 
>> *Assunto:* [obm-l] Dois problemas
>>
>> Boa noite.
>>
>> Quem pode ajudar com esses dois problemas:
>>
>> 1) (Ibero-1992) Para cada inteiro positivo n, seja a_n o último dígito de
>> 1+2+3+...+n. Calcule a_1+a_2+...+a_n.
>>
>> 2) (UK-1997) N é um número inteiro de 4 dígitos não terminado em zero, e
>> R(N) é o número inteiro de 4 dígitos obtido pela reversão dos dígitos de N;
>> por exemplo R(3275)=5723. Determine todos os inteiros N ára os quais
>> R(N)=4N+3.
>>
>> Sds,
>>
>> Rogério
>>
>>
>> --
>> Esta mensagem foi verificada pelo sistema de antivírus e
>> acredita-se estar livre de perigo.
>>
>> --
>> Esta mensagem foi verificada pelo sistema de antivírus e
>> acredita-se estar livre de perigo.
>>
>

-- 
Esta mensagem foi verificada pelo sistema de antiv�rus e
 acredita-se estar livre de perigo.



Re: [obm-l] RE: Dois problemas

2020-04-27 Por tôpico Pedro José
Boa noite!
1) a_n = [n(n+1)]/2 mod 10
Fácilmente se vê que 20 é múltiplo do período minímo pois:
a_19= 0 e a_20=0 logo a_21= a_20+21mod10 =0+1=1=a1.
a_22=a_1+22 mod10 = 3=a_3 É assim sucessivamente.
Então o período é um divisor de 20
p<>1, pois, a_1<>a_2
p<>2, pois, a_1<>a_3
p<>4, pois a_1<>a_5
p<>5, pois a_1<> a_6
p<>10 pois a_1<>a_11
logo p = 20.
Não vem outra forma se calcular a_1 até a_20

a_1=1 e S_1=1
a_2=3  e S_2=4
a_3=6 e S_3=10
a_4=0 e S_4=10
a_5=5 e S_5=15
a_6=1 e S_6= 16
a_7=8 e S_7=24
a_8=6 e S_8=30
a_9=5 e S_9=35
a_10=5 e S_10=40
a_11=6 e S_11=46
a_12=8 e S_12=54
a_13=1 e S_13=55
a_14=5 e S_14=60
a_15=0 e S_15=60
a_16=6 e S_16= 66
a_17=3 e S_17=69
a_18=1 e S_18=70
a_19=0 e S_19=70
a_20=0 e S_20=70
É fácil ver que a cada 20 números a mais a soma dará 70 de novo.,  1992 =
20 x 99 +12; logo S_1992 =99 S_×0+S_12=6984.

2) seja n formado por ABCD, vamos representar a ideia número pela letra
minúscula a, b, c, d e U={a, b, c, d} max(U)=9 e min(U)=0.
a<=2 e a ímpar pois R(n)=DCBA é ímpar.
Então a=1
 4n+3= 4x(a*10^3+b*10^2+c*10+d)+3=
=4d+3 mod10 4d+3=a mod10 ==> d=2 ou d=7, mas d>=4 pois 4n+3= d*10^3 +
c*10^2 + b*10 + a > 4 *10^3. Logo d=7
Logo b=7 ou b=9
pois 4*b +3>30 É b é ímpar pois
DCBA é côngruo 3 mod4
Seja b=7
n é formado por 17C7 e R(n) por 7C71, logo 4*c+3 >=30 e 4c+3=7 mod 10,
portanto não há c que atenda.
Seja b=9
n é formado por 19C7 e R(n) por 7C91, logo 4c+3=9 mod 10 e






Em dom, 26 de abr de 2020 22:56, Julio Mohnsam 
escreveu:

> se n=2019
>
> --
> *De:* owner-ob...@mat.puc-rio.br  em nome de
> Rogério Possi Júnior 
> *Enviado:* domingo, 26 de abril de 2020 18:21
> *Para:* Lista de Olímpiada OBM 
> *Assunto:* [obm-l] Dois problemas
>
> Boa noite.
>
> Quem pode ajudar com esses dois problemas:
>
> 1) (Ibero-1992) Para cada inteiro positivo n, seja a_n o último dígito de
> 1+2+3+...+n. Calcule a_1+a_2+...+a_n.
>
> 2) (UK-1997) N é um número inteiro de 4 dígitos não terminado em zero, e
> R(N) é o número inteiro de 4 dígitos obtido pela reversão dos dígitos de N;
> por exemplo R(3275)=5723. Determine todos os inteiros N ára os quais
> R(N)=4N+3.
>
> Sds,
>
> Rogério
>
>
> --
> Esta mensagem foi verificada pelo sistema de antivírus e
> acredita-se estar livre de perigo.
>
> --
> Esta mensagem foi verificada pelo sistema de antivírus e
> acredita-se estar livre de perigo.
>

-- 
Esta mensagem foi verificada pelo sistema de antiv�rus e
 acredita-se estar livre de perigo.



Re: [obm-l] teoria dos numeros

2020-03-30 Por tôpico Pedro José
Boa noite!

errata:
Ao invés de: 128=2^7 então 2^7| 49^{n} + 81^{n} −2<==> x= 2^7| 49^{n} +
81^{n}=2 mod2^7
128=2^7 então 2^7| 49^{n} + 81^{n} −2<==> x= 49^{n} + 81^{n}=2 mod2^7

Saudações,
PJMS

Em dom., 29 de mar. de 2020 às 14:04, Pedro José 
escreveu:

> Bom dia!
> Prove que 128 divide 49^{n} + 81^{n} −2, para todo n ≥ 1.
> 128=2^7 então 2^7| 49^{n} + 81^{n} −2<==> x= 2^7| 49^{n} + 81^{n}=2 mod2^7
> x= a + b , a= 49^n e b=81^n
> a= (64-15)^n = n(-1)^n*n*64*(15)^(n-1) + (-1)^n*15^n mod2^7; pois, os
> demais termos do binômio de Newton terão o fator (2^6)^m com m>1 que é
> côngruo 0 mod2^7.
> b= (64+17)^n = n*64*17^(n-1) + 17^n mod2^7 pelo mesmo motivo anterior.
> a+b = n*64(17^n-1 +(-1)^(n-1)*15^(n-1)) + 17^n + (-1)^n*15^n =
> (-1)^(n-1)*15^(n-1)) + 17^n + (-1)^n*15^n mod2^; pois a primeira parcela é
> côngrua a 0 mod2^7; já que o termo entre parêntesis é par.
> (16+1)^n= n*16+1 mod2^7 ,pois, (2^4)^m =0 mod2^7 para m>1
> (-1)^n*(16-1)= (-1)^n*[(-1)^(n-1)*n*16+(-1)^n]=-16n +1
> então x = a+b= 2 mod2^7 ==> 2^7 | a+b-2
>
> Saudações,
> PJMS
>
>
>
>
> Em sáb., 28 de mar. de 2020 às 14:05, Israel Meireles Chrisostomo <
> israelmchrisost...@gmail.com> escreveu:
>
>> Eu sei resolver o problema abaixo,porém não sei se é a forma mais simples
>> de se fazer.Vcs poderiam por favor colocar suas soluções nos comentários
>> dessa publicação? O problema é o seguinte:
>> Prove que 128 divide 49^{n} + 81^{n} −2, para todo n ≥ 1.Se possível não
>> use indução, pois eu já estou usando indução.
>>
>> --
>> Israel Meireles Chrisostomo
>>
>> --
>> Esta mensagem foi verificada pelo sistema de antivírus e
>> acredita-se estar livre de perigo.
>
>

-- 
Esta mensagem foi verificada pelo sistema de antiv�rus e
 acredita-se estar livre de perigo.



Re: [obm-l] teoria dos numeros

2020-03-29 Por tôpico Pedro José
Bom dia!
Prove que 128 divide 49^{n} + 81^{n} −2, para todo n ≥ 1.
128=2^7 então 2^7| 49^{n} + 81^{n} −2<==> x= 2^7| 49^{n} + 81^{n}=2 mod2^7
x= a + b , a= 49^n e b=81^n
a= (64-15)^n = n(-1)^n*n*64*(15)^(n-1) + (-1)^n*15^n mod2^7; pois, os
demais termos do binômio de Newton terão o fator (2^6)^m com m>1 que é
côngruo 0 mod2^7.
b= (64+17)^n = n*64*17^(n-1) + 17^n mod2^7 pelo mesmo motivo anterior.
a+b = n*64(17^n-1 +(-1)^(n-1)*15^(n-1)) + 17^n + (-1)^n*15^n =
(-1)^(n-1)*15^(n-1)) + 17^n + (-1)^n*15^n mod2^; pois a primeira parcela é
côngrua a 0 mod2^7; já que o termo entre parêntesis é par.
(16+1)^n= n*16+1 mod2^7 ,pois, (2^4)^m =0 mod2^7 para m>1
(-1)^n*(16-1)= (-1)^n*[(-1)^(n-1)*n*16+(-1)^n]=-16n +1
então x = a+b= 2 mod2^7 ==> 2^7 | a+b-2

Saudações,
PJMS




Em sáb., 28 de mar. de 2020 às 14:05, Israel Meireles Chrisostomo <
israelmchrisost...@gmail.com> escreveu:

> Eu sei resolver o problema abaixo,porém não sei se é a forma mais simples
> de se fazer.Vcs poderiam por favor colocar suas soluções nos comentários
> dessa publicação? O problema é o seguinte:
> Prove que 128 divide 49^{n} + 81^{n} −2, para todo n ≥ 1.Se possível não
> use indução, pois eu já estou usando indução.
>
> --
> Israel Meireles Chrisostomo
>
> --
> Esta mensagem foi verificada pelo sistema de antivírus e
> acredita-se estar livre de perigo.

-- 
Esta mensagem foi verificada pelo sistema de antiv�rus e
 acredita-se estar livre de perigo.



[obm-l] Re: período de dízima

2020-03-24 Por tôpico Pedro José
Bom dia!
Consegui demonstrar que é verdadeira.
Só faltou 2^a||t e 2^b||t ou seja (10,n)=1.

Saudações,
PJMS

Em ter., 10 de mar. de 2020 às 18:39, Pedro José 
escreveu:

> Boa noite!
>
> Alguém poderia provar ou derrubar a conjectura a seguir?
>
> Seja s/t uma fração em que t não divide s e (s,t)=1; seja t=2^a.2^b.n
> O número de algarismos da parte não periódica é o max(a,b) e o número de
> algarismos da parte periódica é a ord 10 mod n.
> Representação decimal.
>
> Saudações,
> PJMS
>

-- 
Esta mensagem foi verificada pelo sistema de antiv�rus e
 acredita-se estar livre de perigo.



[obm-l] Re: [obm-l] Re: [obm-l] Re: [obm-l] Teoria dos números

2020-03-22 Por tôpico Pedro José
Boa tarde!
Perfeita a sua correção.
Quanto ao questionamento, nem tenho formação em matemática, meu sonho é
cursar no IMPA ao me aposentar. Sou pitaqueiro. Ouço um assunto que não
conheço, tento aprendê-lo. Na verdade, gosto de matemática. Talvez seja ela
o "Mundo das ideias", o mundo ideal, a qual Platão se referiu.
Saudações,
PJMS

Em dom, 22 de mar de 2020 12:25, Israel Meireles Chrisostomo <
israelmchrisost...@gmail.com> escreveu:

> Acho q tem uma ´pequena correção no seguinte passo "4k+1. Pegando os
> fatores (4n-1)^2 e (4n+1)^2, teremos que 2^6 |p(n) para qualquer n=4k+1."O
> correto seria "Para n=4k+1.Pegando os fatores (n-1)^2 e (n+1)^2"
>
> Em dom., 22 de mar. de 2020 às 10:14, Israel Meireles Chrisostomo <
> israelmchrisost...@gmail.com> escreveu:
>
>> Primeiramente obrigado pela solução.Mas Pedro, tenho uma pergunta :   o
>> sr. é professor de Matemática?
>>
>> Em dom., 22 de mar. de 2020 às 01:34, Pedro José 
>> escreveu:
>>
>>> Bom dia!
>>> Dei uma mancada.
>>> O expoente de 3 é 3 e não 2.
>>> Retornando às classes mod 3.
>>> Ao último fator é côngruo à (n-1)*n
>>> Para n=3k aparece outro fator e 3^3|p(n), n=3k.
>>> n=3k+1, tenho (n-1)^2 e (n-1), 3^3|p(n), n=3k+1
>>> n=3k+2, tenho(n-2)^2 é (n+1)^2, 3^3|p(n), n=3k+2,
>>> Logo 3^3|p(n) para todo n inteiro.
>>> D>=2^6*3^3*5. Mas D<=2^6*3^3*5, então D=8640
>>> Desculpem-me pelo erro.
>>> Saudações,
>>> PJMS.
>>>
>>>
>>>
>>> Em sáb, 21 de mar de 2020 13:20, Pedro José 
>>> escreveu:
>>>
>>>> Boa tarde!
>>>> Nem carece método numérico.
>>>> Para n=1 ou n=0 ou n=2 temos que qualquer inteiro divide o polinômio
>>>> p(n)=(n-2)^2*(n-1)^2*n^2*(n+1)^2*(4n^2-4n-9)
>>>>
>>>> p(3)=8640
>>>> p(4)=561600 então (p(3),p(4))=8640=2^6*3^3*5.
>>>> Seja D o maior inteiro que divide p(n) para todo n inteiro, D<=8640
>>>> Vamos pegar as classes de equivalência mod 4. Seja k um inteiro.
>>>> Para 4k temos que n^2= 16k^2 e (n-2) é par logo (n-2)^2= 4s^2 com s
>>>> inteiro. Logo 2^6 divide p(n) para qualquer n =4k.
>>>> 4k+1. Pegando os fatores (4n-1)^2 e (4n+1)^2, teremos que 2^6 |p(n)
>>>> para qualquer n=4k+1.
>>>> 4k+2. Pegando (n-2)^2 e n^2, teremos que 2^6|p(n) para qualquer n=4k+2
>>>> 4k+3, pegando os mesmos fatores de 4k+1, 2^6|p(n) para n=4k+3.
>>>> Portanto 2^6|p(n) para qualquer inteiro
>>>> Agora classes de equivalência mod 3
>>>> 3k, pegando n^2, 3^2|p(n) para n=3k
>>>> 3k+1, pegando (n-1)^2; 3^2| p(n), n=3k+1
>>>> 3k+2, pegando (n-2)^2, 3^2| p(n), n=3k+2
>>>> Daí 3^2|p(n) para qualquer n inteiro.
>>>> Classes de equivalência mod 5.
>>>> 5k, n^2 , 5 |p(n), n =5
>>>> 5k +1, (n-1)^2, 5|p(n), n=5k+1
>>>> 5k+2, (n-2)^2, 5|p(n), n=5k+2
>>>> 5k+3, (4n^2-4n-9)=(100k^2-100k+15)
>>>> 5|p(n), n=5k+3
>>>> 5k+4, (n+1)^2, 5|p(n) , n=5k+4.
>>>> Então 5|p(n) para todo inteiro
>>>> D>=2^6*3^2×*5
>>>> Mas D<=2^6*3^2*5, logo D=8640
>>>>
>>>> Saudações,
>>>> PJMS
>>>>
>>>> Em sáb, 21 de mar de 2020 04:39, Pedro José 
>>>> escreveu:
>>>>
>>>>> Bom dia!
>>>>> Falta de novo, em seu questionamento, informar que n é inteiro ou
>>>>> natural e colocar a condição para qualquer valor de n. Chamando o 
>>>>> polinômio
>>>>> de p(n)
>>>>> Para n=0, 1 ou 2, qualquer inteiro divide.
>>>>> Faria mdc(p(3),p(4))= A1
>>>>> Se der "pequeno", com poucos fatores primos e expoentes pequenos. Paro
>>>>> em A1, se não.
>>>>> (p(5),A1)=A2 uso o mesmo critério de parar
>>>>> (p(6),A2)=A3 até parar em:
>>>>> Ai=(p(i+3),A(i-1)).
>>>>> Aí faço o polinômio módfi^xi, onde fi é um fator primo de Aí e xi seu
>>>>> expoente. verifico se para cada resíduon= 1, 2...fi^n-1 se P(n)=0 mod 
>>>>> fi^si
>>>>> Se falhar diminuto xi em 1 e repito o teste para todos resíduos de
>>>>> fi^(xi-1)-1 até um dado xki em que todos os p(resíduos) foram equivalente 
>>>>> a
>>>>> zero módulo fi^xki ou quando fizer para o expoente 1  e não zerar para
>>>>> todos resíduos de fi, quando o fator será descartado.
>>>>> Depois repito para cada fator primo f e seu respectivo expoe

[obm-l] Re: [obm-l] Teoria dos números

2020-03-21 Por tôpico Pedro José
Bom dia!
Dei uma mancada.
O expoente de 3 é 3 e não 2.
Retornando às classes mod 3.
Ao último fator é côngruo à (n-1)*n
Para n=3k aparece outro fator e 3^3|p(n), n=3k.
n=3k+1, tenho (n-1)^2 e (n-1), 3^3|p(n), n=3k+1
n=3k+2, tenho(n-2)^2 é (n+1)^2, 3^3|p(n), n=3k+2,
Logo 3^3|p(n) para todo n inteiro.
D>=2^6*3^3*5. Mas D<=2^6*3^3*5, então D=8640
Desculpem-me pelo erro.
Saudações,
PJMS.



Em sáb, 21 de mar de 2020 13:20, Pedro José  escreveu:

> Boa tarde!
> Nem carece método numérico.
> Para n=1 ou n=0 ou n=2 temos que qualquer inteiro divide o polinômio
> p(n)=(n-2)^2*(n-1)^2*n^2*(n+1)^2*(4n^2-4n-9)
>
> p(3)=8640
> p(4)=561600 então (p(3),p(4))=8640=2^6*3^3*5.
> Seja D o maior inteiro que divide p(n) para todo n inteiro, D<=8640
> Vamos pegar as classes de equivalência mod 4. Seja k um inteiro.
> Para 4k temos que n^2= 16k^2 e (n-2) é par logo (n-2)^2= 4s^2 com s
> inteiro. Logo 2^6 divide p(n) para qualquer n =4k.
> 4k+1. Pegando os fatores (4n-1)^2 e (4n+1)^2, teremos que 2^6 |p(n) para
> qualquer n=4k+1.
> 4k+2. Pegando (n-2)^2 e n^2, teremos que 2^6|p(n) para qualquer n=4k+2
> 4k+3, pegando os mesmos fatores de 4k+1, 2^6|p(n) para n=4k+3.
> Portanto 2^6|p(n) para qualquer inteiro
> Agora classes de equivalência mod 3
> 3k, pegando n^2, 3^2|p(n) para n=3k
> 3k+1, pegando (n-1)^2; 3^2| p(n), n=3k+1
> 3k+2, pegando (n-2)^2, 3^2| p(n), n=3k+2
> Daí 3^2|p(n) para qualquer n inteiro.
> Classes de equivalência mod 5.
> 5k, n^2 , 5 |p(n), n =5
> 5k +1, (n-1)^2, 5|p(n), n=5k+1
> 5k+2, (n-2)^2, 5|p(n), n=5k+2
> 5k+3, (4n^2-4n-9)=(100k^2-100k+15)
> 5|p(n), n=5k+3
> 5k+4, (n+1)^2, 5|p(n) , n=5k+4.
> Então 5|p(n) para todo inteiro
> D>=2^6*3^2×*5
> Mas D<=2^6*3^2*5, logo D=8640
>
> Saudações,
> PJMS
>
> Em sáb, 21 de mar de 2020 04:39, Pedro José 
> escreveu:
>
>> Bom dia!
>> Falta de novo, em seu questionamento, informar que n é inteiro ou natural
>> e colocar a condição para qualquer valor de n. Chamando o polinômio de p(n)
>> Para n=0, 1 ou 2, qualquer inteiro divide.
>> Faria mdc(p(3),p(4))= A1
>> Se der "pequeno", com poucos fatores primos e expoentes pequenos. Paro em
>> A1, se não.
>> (p(5),A1)=A2 uso o mesmo critério de parar
>> (p(6),A2)=A3 até parar em:
>> Ai=(p(i+3),A(i-1)).
>> Aí faço o polinômio módfi^xi, onde fi é um fator primo de Aí e xi seu
>> expoente. verifico se para cada resíduon= 1, 2...fi^n-1 se P(n)=0 mod fi^si
>> Se falhar diminuto xi em 1 e repito o teste para todos resíduos de
>> fi^(xi-1)-1 até um dado xki em que todos os p(resíduos) foram equivalente a
>> zero módulo fi^xki ou quando fizer para o expoente 1  e não zerar para
>> todos resíduos de fi, quando o fator será descartado.
>> Depois repito para cada fator primo f e seu respectivo expoente.
>> Ao final D = Produtório de cada fator fi elevado ao expoente xki que
>> zerou p(n) mod fi^xki para todos os resíduos, descartando os fí em que xji
>> chegou a 1 e não atendeu ou considerando nesse caso xki=0.
>>
>> Mas resolveria por método numérico.
>> Depois poste sua solução.
>>
>> Saudações,
>> PJMS.
>>
>>
>>
>>
>> Em sex, 20 de mar de 2020 12:42, Israel Meireles Chrisostomo <
>> israelmchrisost...@gmail.com> escreveu:
>>
>>> Qual o maior inteiro que divide (n - 2)^2 (n - 1)^2 n^2 (n + 1)^2 (4 n^2
>>> - 4 n - 9))?
>>> Eu sei resolver esse problema com meu algoritmo, porém gostaria de saber
>>> como os colegas o resolveriam.
>>> --
>>> Israel Meireles Chrisostomo
>>>
>>> --
>>> Esta mensagem foi verificada pelo sistema de antivírus e
>>> acredita-se estar livre de perigo.
>>
>>

-- 
Esta mensagem foi verificada pelo sistema de antiv�rus e
 acredita-se estar livre de perigo.



[obm-l] Re: [obm-l] Teoria dos números

2020-03-21 Por tôpico Pedro José
Boa tarde!
Nem carece método numérico.
Para n=1 ou n=0 ou n=2 temos que qualquer inteiro divide o polinômio
p(n)=(n-2)^2*(n-1)^2*n^2*(n+1)^2*(4n^2-4n-9)

p(3)=8640
p(4)=561600 então (p(3),p(4))=8640=2^6*3^3*5.
Seja D o maior inteiro que divide p(n) para todo n inteiro, D<=8640
Vamos pegar as classes de equivalência mod 4. Seja k um inteiro.
Para 4k temos que n^2= 16k^2 e (n-2) é par logo (n-2)^2= 4s^2 com s
inteiro. Logo 2^6 divide p(n) para qualquer n =4k.
4k+1. Pegando os fatores (4n-1)^2 e (4n+1)^2, teremos que 2^6 |p(n) para
qualquer n=4k+1.
4k+2. Pegando (n-2)^2 e n^2, teremos que 2^6|p(n) para qualquer n=4k+2
4k+3, pegando os mesmos fatores de 4k+1, 2^6|p(n) para n=4k+3.
Portanto 2^6|p(n) para qualquer inteiro
Agora classes de equivalência mod 3
3k, pegando n^2, 3^2|p(n) para n=3k
3k+1, pegando (n-1)^2; 3^2| p(n), n=3k+1
3k+2, pegando (n-2)^2, 3^2| p(n), n=3k+2
Daí 3^2|p(n) para qualquer n inteiro.
Classes de equivalência mod 5.
5k, n^2 , 5 |p(n), n =5
5k +1, (n-1)^2, 5|p(n), n=5k+1
5k+2, (n-2)^2, 5|p(n), n=5k+2
5k+3, (4n^2-4n-9)=(100k^2-100k+15)
5|p(n), n=5k+3
5k+4, (n+1)^2, 5|p(n) , n=5k+4.
Então 5|p(n) para todo inteiro
D>=2^6*3^2×*5
Mas D<=2^6*3^2*5, logo D=8640

Saudações,
PJMS

Em sáb, 21 de mar de 2020 04:39, Pedro José  escreveu:

> Bom dia!
> Falta de novo, em seu questionamento, informar que n é inteiro ou natural
> e colocar a condição para qualquer valor de n. Chamando o polinômio de p(n)
> Para n=0, 1 ou 2, qualquer inteiro divide.
> Faria mdc(p(3),p(4))= A1
> Se der "pequeno", com poucos fatores primos e expoentes pequenos. Paro em
> A1, se não.
> (p(5),A1)=A2 uso o mesmo critério de parar
> (p(6),A2)=A3 até parar em:
> Ai=(p(i+3),A(i-1)).
> Aí faço o polinômio módfi^xi, onde fi é um fator primo de Aí e xi seu
> expoente. verifico se para cada resíduon= 1, 2...fi^n-1 se P(n)=0 mod fi^si
> Se falhar diminuto xi em 1 e repito o teste para todos resíduos de
> fi^(xi-1)-1 até um dado xki em que todos os p(resíduos) foram equivalente a
> zero módulo fi^xki ou quando fizer para o expoente 1  e não zerar para
> todos resíduos de fi, quando o fator será descartado.
> Depois repito para cada fator primo f e seu respectivo expoente.
> Ao final D = Produtório de cada fator fi elevado ao expoente xki que zerou
> p(n) mod fi^xki para todos os resíduos, descartando os fí em que xji chegou
> a 1 e não atendeu ou considerando nesse caso xki=0.
>
> Mas resolveria por método numérico.
> Depois poste sua solução.
>
> Saudações,
> PJMS.
>
>
>
>
> Em sex, 20 de mar de 2020 12:42, Israel Meireles Chrisostomo <
> israelmchrisost...@gmail.com> escreveu:
>
>> Qual o maior inteiro que divide (n - 2)^2 (n - 1)^2 n^2 (n + 1)^2 (4 n^2
>> - 4 n - 9))?
>> Eu sei resolver esse problema com meu algoritmo, porém gostaria de saber
>> como os colegas o resolveriam.
>> --
>> Israel Meireles Chrisostomo
>>
>> --
>> Esta mensagem foi verificada pelo sistema de antivírus e
>> acredita-se estar livre de perigo.
>
>

-- 
Esta mensagem foi verificada pelo sistema de antiv�rus e
 acredita-se estar livre de perigo.



[obm-l] Re: [obm-l] Teoria dos números

2020-03-21 Por tôpico Pedro José
Bom dia!
Falta de novo, em seu questionamento, informar que n é inteiro ou natural e
colocar a condição para qualquer valor de n. Chamando o polinômio de p(n)
Para n=0, 1 ou 2, qualquer inteiro divide.
Faria mdc(p(3),p(4))= A1
Se der "pequeno", com poucos fatores primos e expoentes pequenos. Paro em
A1, se não.
(p(5),A1)=A2 uso o mesmo critério de parar
(p(6),A2)=A3 até parar em:
Ai=(p(i+3),A(i-1)).
Aí faço o polinômio módfi^xi, onde fi é um fator primo de Aí e xi seu
expoente. verifico se para cada resíduon= 1, 2...fi^n-1 se P(n)=0 mod fi^si
Se falhar diminuto xi em 1 e repito o teste para todos resíduos de
fi^(xi-1)-1 até um dado xki em que todos os p(resíduos) foram equivalente a
zero módulo fi^xki ou quando fizer para o expoente 1  e não zerar para
todos resíduos de fi, quando o fator será descartado.
Depois repito para cada fator primo f e seu respectivo expoente.
Ao final D = Produtório de cada fator fi elevado ao expoente xki que zerou
p(n) mod fi^xki para todos os resíduos, descartando os fí em que xji chegou
a 1 e não atendeu ou considerando nesse caso xki=0.

Mas resolveria por método numérico.
Depois poste sua solução.

Saudações,
PJMS.




Em sex, 20 de mar de 2020 12:42, Israel Meireles Chrisostomo <
israelmchrisost...@gmail.com> escreveu:

> Qual o maior inteiro que divide (n - 2)^2 (n - 1)^2 n^2 (n + 1)^2 (4 n^2 -
> 4 n - 9))?
> Eu sei resolver esse problema com meu algoritmo, porém gostaria de saber
> como os colegas o resolveriam.
> --
> Israel Meireles Chrisostomo
>
> --
> Esta mensagem foi verificada pelo sistema de antivírus e
> acredita-se estar livre de perigo.

-- 
Esta mensagem foi verificada pelo sistema de antiv�rus e
 acredita-se estar livre de perigo.



Re: [obm-l] Inteiros (divisibilidade)

2020-03-18 Por tôpico Pedro José
Bom dia!
Caso contrário fica simples.
b=-1 ==> a= -1 (-1,-1)
b=0 ou b=-2  ==> qualquer a
a=-1 ==> b qualquer
Para outros casos: a+1 é múltiplo de b+1
Generalizando:  |a+1|= |k(b+1)| com k inteiro

Em qua., 18 de mar. de 2020 às 09:04, Pedro José 
escreveu:

> Bom dia!
> Não há outra restrição?
> É igual perguntar quais os pares de inteiros (x,y) tais que x|y, com x=b+1
> e y=a+1.
>
> Saudações,
> PJMS
>
> Em qua., 18 de mar. de 2020 às 08:51, marcone augusto araújo borges <
> marconeborge...@hotmail.com> escreveu:
>
>> Determine todos os pares de inteiros a e b tais que a divide b+1 e b
>> divide a+1
>> Desde já agradeço
>> --
>> Esta mensagem foi verificada pelo sistema de antivírus e
>> acredita-se estar livre de perigo.
>>
>

-- 
Esta mensagem foi verificada pelo sistema de antiv�rus e
 acredita-se estar livre de perigo.



Re: [obm-l] Inteiros (divisibilidade)

2020-03-18 Por tôpico Pedro José
Bom dia!
Não há outra restrição?
É igual perguntar quais os pares de inteiros (x,y) tais que x|y, com x=b+1
e y=a+1.

Saudações,
PJMS

Em qua., 18 de mar. de 2020 às 08:51, marcone augusto araújo borges <
marconeborge...@hotmail.com> escreveu:

> Determine todos os pares de inteiros a e b tais que a divide b+1 e b
> divide a+1
> Desde já agradeço
> --
> Esta mensagem foi verificada pelo sistema de antivírus e
> acredita-se estar livre de perigo.
>

-- 
Esta mensagem foi verificada pelo sistema de antiv�rus e
 acredita-se estar livre de perigo.



Re: [obm-l] Problema

2020-03-17 Por tôpico Pedro José
Boa noite!
Você já formulou esse problema em set/2019 e Daniel Jelin apresentou uma
bela solução.
Saudações,
PJMS

Em ter, 17 de mar de 2020 19:26,  escreveu:

> Problema
> Um mágico e seu assistente realizam um truque da maneira seguinte. Existem
> 12 caixas vazias e fechadas, colocadas em fila. O mágico sai da sala e uma
> pessoa do público escolhe duas caixas e esconde em cada uma delas uma
> moeda, deixando a fila de caixas da mesma forma como era, mas o assistente
> sabe quais são as duas caixas que têm as moedas. O mágico retorna para a
> sala e o assistente escolhe uma caixa que ele sabe que está vazia. Das
> restantes, o mágico então escolhe quatro caixas que são abertas
> simultaneamente. O objetivo do mágico é que, entre essas quatro caixas,
> duas contenham as moedas.
> Desenvolva um método que permita que o mágico e seu assistente realizem a
> mágica com sucesso
>
>
> --
> Esta mensagem foi verificada pelo sistema de antivírus e
> acredita-se estar livre de perigo.Booa
>

-- 
Esta mensagem foi verificada pelo sistema de antiv�rus e
 acredita-se estar livre de perigo.



[obm-l] Re: [obm-l] Re: [obm-l] Re: [obm-l] Re: [obm-l] Re: [obm-l] Re: [obm-l] Teoria dos números

2020-03-17 Por tôpico Pedro José
Boa noite!
Aí, como dizia minha falecida vó, são outros quinhentos.
Como nas propostas anteriores n era natural. Vamos seguir nessa linha, se
não for reformule o problema.
Seja f(n)=   n (427 - 90n - 70n^2 + 45n^3 + 18n^4)
f(0)=0 qualquer natural divide, portanto, é indiferente.
f(1)= 330
f(2)= 1230
É fácil verificar que mdc(330,1230)=30 então D<=30, onde D é o máximo
inteiro que divide f(n) para todo n natural.
f(n) = 7n +5n^4 + 8 n^5 mod 10.
f(0)=0 mod10
f(1)= 20 = 0 mod10
f(2)= 350= 0 mod10
f(3)= 2370 = 0 mod10
f(4)= 9500 = 0 mod10
f(5)= 28160 = 0 mod10
f(6)=68730 = 0 mod10
f(7)=146510 = 0 mod10
f(8)=282680 = 0 mod10
f(9)=505260 = 0 mod10
logo 10 | f(n) para qualquer n natural.

f(n) = n -n^3 mod 3
f(0) = 0 mod 3
f(1) = 0 mod 3
f(2)= -6 = 0 mod 3
logo 3| f(n) para todo n natural
então D = 30.

Saudações,
PJMS



Em ter., 17 de mar. de 2020 às 11:57, Israel Meireles Chrisostomo <
israelmchrisost...@gmail.com> escreveu:

> Sim é isso q eu quis dizer
>
> Em ter, 17 de mar de 2020 11:12, Carlos Gustavo Tamm de Araujo Moreira <
> g...@impa.br> escreveu:
>
>> Acho que a pergunta deve ser qual é o maior inteiro positivo que divide
>> essa expressão para todo valor de n ao mesmo tempo.
>>
>> On Tue, Mar 17, 2020 at 6:58 AM Pedro José  wrote:
>>
>>> Bom dia!
>>> Se você considerar a expressão n(427-90n-70n^2+45n^3+18n^4)
>>> D=|n(427-90n-70n^2+45n^3+18n^4)|
>>> Por exemplo, n=1
>>> D=330.
>>> Agora se liberar n para variar D tende a oo.
>>>
>>> Se n for raiz da expressão, também tende a oi, pois qualquer inteiro
>>> divide 0.
>>>
>>>
>>> Em seg, 16 de mar de 2020 22:16, Israel Meireles Chrisostomo <
>>> israelmchrisost...@gmail.com> escreveu:
>>>
>>>> não entendi
>>>>
>>>> Em seg., 16 de mar. de 2020 às 22:01, Pedro José 
>>>> escreveu:
>>>>
>>>>> Para um dado n é o módulo do valor da expressão.
>>>>>
>>>>> Em seg, 16 de mar de 2020 21:49, Pedro José 
>>>>> escreveu:
>>>>>
>>>>>> Boa noite!
>>>>>> O módulo dessa expressão tende a oo. Não existe máximo.
>>>>>> Saudações,
>>>>>> PJMS
>>>>>>
>>>>>> Em seg, 16 de mar de 2020 20:36, Israel Meireles Chrisostomo <
>>>>>> israelmchrisost...@gmail.com> escreveu:
>>>>>>
>>>>>>> Qual é o maior inteiro que divide  n (427 - 90n - 70n^2 + 45n^3 +
>>>>>>> 18n^4)?
>>>>>>>
>>>>>>> --
>>>>>>> Israel Meireles Chrisostomo
>>>>>>>
>>>>>>> --
>>>>>>> Esta mensagem foi verificada pelo sistema de antivírus e
>>>>>>> acredita-se estar livre de perigo.
>>>>>>
>>>>>>
>>>>> --
>>>>> Esta mensagem foi verificada pelo sistema de antivírus e
>>>>> acredita-se estar livre de perigo.
>>>>
>>>>
>>>>
>>>> --
>>>> Israel Meireles Chrisostomo
>>>>
>>>> --
>>>> Esta mensagem foi verificada pelo sistema de antivírus e
>>>> acredita-se estar livre de perigo.
>>>
>>>
>>> --
>>> Esta mensagem foi verificada pelo sistema de antivírus e
>>> acredita-se estar livre de perigo.
>>
>>
>> --
>> Esta mensagem foi verificada pelo sistema de antivírus e
>> acredita-se estar livre de perigo.
>
>
> --
> Esta mensagem foi verificada pelo sistema de antivírus e
> acredita-se estar livre de perigo.

-- 
Esta mensagem foi verificada pelo sistema de antiv�rus e
 acredita-se estar livre de perigo.



[obm-l] Re: [obm-l] Re: [obm-l] Re: [obm-l] Teoria dos números

2020-03-17 Por tôpico Pedro José
Bom dia!
Se você considerar a expressão n(427-90n-70n^2+45n^3+18n^4)
D=|n(427-90n-70n^2+45n^3+18n^4)|
Por exemplo, n=1
D=330.
Agora se liberar n para variar D tende a oo.

Se n for raiz da expressão, também tende a oi, pois qualquer inteiro divide
0.


Em seg, 16 de mar de 2020 22:16, Israel Meireles Chrisostomo <
israelmchrisost...@gmail.com> escreveu:

> não entendi
>
> Em seg., 16 de mar. de 2020 às 22:01, Pedro José 
> escreveu:
>
>> Para um dado n é o módulo do valor da expressão.
>>
>> Em seg, 16 de mar de 2020 21:49, Pedro José 
>> escreveu:
>>
>>> Boa noite!
>>> O módulo dessa expressão tende a oo. Não existe máximo.
>>> Saudações,
>>> PJMS
>>>
>>> Em seg, 16 de mar de 2020 20:36, Israel Meireles Chrisostomo <
>>> israelmchrisost...@gmail.com> escreveu:
>>>
>>>> Qual é o maior inteiro que divide  n (427 - 90n - 70n^2 + 45n^3 +
>>>> 18n^4)?
>>>>
>>>> --
>>>> Israel Meireles Chrisostomo
>>>>
>>>> --
>>>> Esta mensagem foi verificada pelo sistema de antivírus e
>>>> acredita-se estar livre de perigo.
>>>
>>>
>> --
>> Esta mensagem foi verificada pelo sistema de antivírus e
>> acredita-se estar livre de perigo.
>
>
>
> --
> Israel Meireles Chrisostomo
>
> --
> Esta mensagem foi verificada pelo sistema de antivírus e
> acredita-se estar livre de perigo.

-- 
Esta mensagem foi verificada pelo sistema de antiv�rus e
 acredita-se estar livre de perigo.



[obm-l] Re: [obm-l] Teoria dos números

2020-03-16 Por tôpico Pedro José
Boa noite!
O módulo dessa expressão tende a oo. Não existe máximo.
Saudações,
PJMS

Em seg, 16 de mar de 2020 20:36, Israel Meireles Chrisostomo <
israelmchrisost...@gmail.com> escreveu:

> Qual é o maior inteiro que divide  n (427 - 90n - 70n^2 + 45n^3 + 18n^4)?
>
> --
> Israel Meireles Chrisostomo
>
> --
> Esta mensagem foi verificada pelo sistema de antivírus e
> acredita-se estar livre de perigo.

-- 
Esta mensagem foi verificada pelo sistema de antiv�rus e
 acredita-se estar livre de perigo.



[obm-l] Re: [obm-l] Teoria dos números

2020-03-16 Por tôpico Pedro José
Para um dado n é o módulo do valor da expressão.

Em seg, 16 de mar de 2020 21:49, Pedro José  escreveu:

> Boa noite!
> O módulo dessa expressão tende a oo. Não existe máximo.
> Saudações,
> PJMS
>
> Em seg, 16 de mar de 2020 20:36, Israel Meireles Chrisostomo <
> israelmchrisost...@gmail.com> escreveu:
>
>> Qual é o maior inteiro que divide  n (427 - 90n - 70n^2 + 45n^3 + 18n^4)?
>>
>> --
>> Israel Meireles Chrisostomo
>>
>> --
>> Esta mensagem foi verificada pelo sistema de antivírus e
>> acredita-se estar livre de perigo.
>
>

-- 
Esta mensagem foi verificada pelo sistema de antiv�rus e
 acredita-se estar livre de perigo.



[obm-l] Re: [obm-l] Teoria dos números

2020-03-16 Por tôpico Pedro José
Boa tarde!

Difícil generalizar. Mas consegui dois valores que não zeram a expressão
(soluções triviais), a duras penas, n=32 e n=43.
Vou continuar pensando no assunto.

Saudações,
PJMS


Em dom., 15 de mar. de 2020 às 18:48, Pedro José 
escreveu:

> Boa tarde!
> Faltou um contraexemplo.
> n=5
> 3^2*4^2*5^2*6^2*71 não é múltiplo de 11 nem de 37.
>
> Saudações,
> PJMS
>
> Em sáb, 14 de mar de 2020 19:47, Pedro José 
> escreveu:
>
>> Boa noite!
>> Creio que a pergunta correta seria, para que valores de n natural...
>> 8140=2^2*5*11*37. Então a solução só se dará para um subconjunto dos
>> naturais diferente de|N.
>>
>> Saudações,
>> PJMS
>>
>>
>> Em sex, 13 de mar de 2020 20:05, Israel Meireles Chrisostomo <
>> israelmchrisost...@gmail.com> escreveu:
>>
>>>   Dado n natural verifique se a expressão
>>>  (n − 2)² (n − 1)²n² (n + 1)² (4n²− 4n − 9)/8140 é um número inteiro
>>>
>>> --
>>> Israel Meireles Chrisostomo
>>>
>>> --
>>> Esta mensagem foi verificada pelo sistema de antivírus e
>>> acredita-se estar livre de perigo.
>>
>>

-- 
Esta mensagem foi verificada pelo sistema de antiv�rus e
 acredita-se estar livre de perigo.



[obm-l] Re: [obm-l] Teoria dos números

2020-03-15 Por tôpico Pedro José
Boa tarde!
Faltou um contraexemplo.
n=5
3^2*4^2*5^2*6^2*71 não é múltiplo de 11 nem de 37.

Saudações,
PJMS

Em sáb, 14 de mar de 2020 19:47, Pedro José  escreveu:

> Boa noite!
> Creio que a pergunta correta seria, para que valores de n natural...
> 8140=2^2*5*11*37. Então a solução só se dará para um subconjunto dos
> naturais diferente de|N.
>
> Saudações,
> PJMS
>
>
> Em sex, 13 de mar de 2020 20:05, Israel Meireles Chrisostomo <
> israelmchrisost...@gmail.com> escreveu:
>
>>   Dado n natural verifique se a expressão
>>  (n − 2)² (n − 1)²n² (n + 1)² (4n²− 4n − 9)/8140 é um número inteiro
>>
>> --
>> Israel Meireles Chrisostomo
>>
>> --
>> Esta mensagem foi verificada pelo sistema de antivírus e
>> acredita-se estar livre de perigo.
>
>

-- 
Esta mensagem foi verificada pelo sistema de antiv�rus e
 acredita-se estar livre de perigo.



[obm-l] Re: [obm-l] Teoria dos números

2020-03-14 Por tôpico Pedro José
Boa noite!
Creio que a pergunta correta seria, para que valores de n natural...
8140=2^2*5*11*37. Então a solução só se dará para um subconjunto dos
naturais diferente de|N.

Saudações,
PJMS


Em sex, 13 de mar de 2020 20:05, Israel Meireles Chrisostomo <
israelmchrisost...@gmail.com> escreveu:

>   Dado n natural verifique se a expressão
>  (n − 2)² (n − 1)²n² (n + 1)² (4n²− 4n − 9)/8140 é um número inteiro
>
> --
> Israel Meireles Chrisostomo
>
> --
> Esta mensagem foi verificada pelo sistema de antivírus e
> acredita-se estar livre de perigo.

-- 
Esta mensagem foi verificada pelo sistema de antiv�rus e
 acredita-se estar livre de perigo.



[obm-l] Soma surpreendentemente inteira

2020-03-11 Por tôpico Pedro Cardoso
Olá, amigos.

Gostaria de ajuda para calcular a segunte soma:

Soma com n variando de 1 a 7 de
3/(cos(24n)-1)

Com o argumento do cos em graus

Aparentemente essa soma é 56, não consegui entender porque

-- 
Esta mensagem foi verificada pelo sistema de antiv�rus e
 acredita-se estar livre de perigo.



[obm-l] período de dízima

2020-03-10 Por tôpico Pedro José
Boa noite!

Alguém poderia provar ou derrubar a conjectura a seguir?

Seja s/t uma fração em que t não divide s e (s,t)=1; seja t=2^a.2^b.n
O número de algarismos da parte não periódica é o max(a,b) e o número de
algarismos da parte periódica é a ord 10 mod n.
Representação decimal.

Saudações,
PJMS

-- 
Esta mensagem foi verificada pelo sistema de antiv�rus e
 acredita-se estar livre de perigo.



[obm-l] Re: [obm-l] Re: [obm-l] Re: [obm-l] Ajuda com dízima

2020-03-09 Por tôpico Pedro José
Boa noite!
Errata da nota anterior independente de m e não de m, supondo (m,n)=1 e m/n
não inteiro.
Outro ponto não há necessidade a verificação de se o proposto vale para
quando n for múltiplo de 2 ou de 10, pois a ordem m mod n só existe se
(10,n)=1. Foi bobagem só ter aventado a possibilidade.
não coloquei como cheguei a conclusão de que era ordem 10 mod n, pois achei
bem intuitivo. Mas na hora que fui mostrar, achei complicado o que julgara
fácil. Mas quanto a isso estou seguro.
Para (n,m)=1 e (n,10)=1 e n/m não inteiro.
Se m>n pode-se representar por uma fração q j/n com q, j e n inteiros e
(j,n)=1 pois m=qn+j e se d<>1 divide n e j então d|m pois m é uma Z
combinação linear de j e n. Absurdo pois(m,n)=1 por hipótese.
Então sem perda de generalidade podemos só trabalhar para o caso m=2, está correta.

Saudações,
PJMS


Em dom, 8 de mar de 2020 16:09, Pedro José  escreveu:

> Boa tarde!
> Douglas,
> Não creio, no meu entendimento 3^2003 é o número de algarismos da dízima
> pois, é a ordem 10 módulo 3^2005.
> 1/3^2005 tem uma montoeira de algarismos zeros no início do período o que
> não acontece em 3^2005.
> O número de algarismos do período de uma dízima m/n, pelo menos quando n
> não  é múltiplo dos primos 2 e 5 é ord10mod n e independe de n. Nao
> verifiquei se vale sem a restriçao.
> Por exemplo o período de 1/7 é 142857 e ord 10 mod 7 = 6.
> Se aquele fosse o período da dízima bastaria fazer n =[log10 (3^2003)+1]
> onde colchetes representam parte inteira..
> Minha dúvida está na prova por absurdo, que ord 10 mod 3^n= 3^(n-2).
>
> Saudações,
> PJMS
>
>
>
> Em dom, 8 de mar de 2020 11:31, Prof. Douglas Oliveira <
> profdouglaso.del...@gmail.com> escreveu:
>
>> 3^2003 é o período certo??, o número de dígitos disso que seria a
>> pergunta.
>> 
>>
>> Douglas oliveira
>>
>> Em dom, 8 de mar de 2020 11:13, Prof. Douglas Oliveira <
>> profdouglaso.del...@gmail.com> escreveu:
>>
>>> Olá Pedro, primeiramente muito obrigado pela sua solução, eu dei uma
>>> olhada rápida e acredito estar correta. Estarei olhando com mais calma,
>>> assim que tiver um tempinho.
>>>
>>> Douglas Oliveira.
>>>
>>> Em dom, 8 de mar de 2020 11:05, Pedro José 
>>> escreveu:
>>>
>>>> Bom dia!
>>>> Não compreendi o porquê dessa questão ter sido vilipendiada. Não sou
>>>> matemático, sou pitaqueiro, ouço falar em inteiros de Gauss vou atrás, de
>>>> espaço fibrado idem, equações de Pell idem..., o que não consigo aprender
>>>> fica para o futuro. Quando me aposentar  cursar uma faculdade de
>>>> matemática. Portanto, nem tudo que resolvo me dá segurança. Reforço, alguém
>>>> poderia me informar se está correto?
>>>> Saudações,
>>>> PJMS.
>>>>
>>>> Em ter, 3 de mar de 2020 12:03, Pedro José 
>>>> escreveu:
>>>>
>>>>> Boa tarde!
>>>>> Não me senti muito seguro na resposta. Está correto?
>>>>>
>>>>> Saudações,
>>>>> PJMS
>>>>>
>>>>> Em seg., 2 de mar. de 2020 às 23:27, Pedro José 
>>>>> escreveu:
>>>>>
>>>>>> Boa noite!
>>>>>> Creio ter conseguido.
>>>>>> Seja k o número de algarismos do período de 1/3^2005. Como (3,10)=1
>>>>>> então k é a ordem 10 mod 3^2005.
>>>>>> 3^(n-2)|| 3^(n-2); (|| significa divide exatamente) e 3^2||10-1 então
>>>>>> pelo lema de Hensel 3^n||10^(3^(n-2))-1 para n>=2.(i)
>>>>>> Então 10^(3^(n-2))= 1 mod 3^n logo ord 10 mod 3^n | 3^(n-2) Se
>>>>>> x<>3^(n-2) absurdo; pois, teria que ser 3^k com k>>>>> e por (i) 3^(k+2)||10^(3^k)-1 e k+2>>>>> ord 10 mod 3^2005 =3^2003
>>>>>> 3^2003 algarismos
>>>>>> Saudações,
>>>>>> PJMS
>>>>>>
>>>>>> Em sáb, 29 de fev de 2020 16:13, Pedro José 
>>>>>> escreveu:
>>>>>>
>>>>>>> Boa tarde!
>>>>>>> 3^2005 e não 10^2005.
>>>>>>>
>>>>>>> Em sex, 28 de fev de 2020 16:06, Pedro José 
>>>>>>> escreveu:
>>>>>>>
>>>>>>>> Boa tarde!
>>>>>>>> Questão complicada.
>>>>>>>> Como (3^2005; 10) =1, o número de dígitos x deve ser a ordem de 10
>>>>>>>> mod 10^2005. Portanto x | 2*3^2004.
>>>>>>>> Se 10 fosse uma raiz primitiva de 3^2005 aí da

[obm-l] Re: [obm-l] Re: [obm-l] Re: [obm-l] Ajuda com dízima

2020-03-08 Por tôpico Pedro José
Boa tarde!
Douglas,
Não creio, no meu entendimento 3^2003 é o número de algarismos da dízima
pois, é a ordem 10 módulo 3^2005.
1/3^2005 tem uma montoeira de algarismos zeros no início do período o que
não acontece em 3^2005.
O número de algarismos do período de uma dízima m/n, pelo menos quando n
não  é múltiplo dos primos 2 e 5 é ord10mod n e independe de n. Nao
verifiquei se vale sem a restriçao.
Por exemplo o período de 1/7 é 142857 e ord 10 mod 7 = 6.
Se aquele fosse o período da dízima bastaria fazer n =[log10 (3^2003)+1]
onde colchetes representam parte inteira..
Minha dúvida está na prova por absurdo, que ord 10 mod 3^n= 3^(n-2).

Saudações,
PJMS



Em dom, 8 de mar de 2020 11:31, Prof. Douglas Oliveira <
profdouglaso.del...@gmail.com> escreveu:

> 3^2003 é o período certo??, o número de dígitos disso que seria a pergunta.
> 
>
> Douglas oliveira
>
> Em dom, 8 de mar de 2020 11:13, Prof. Douglas Oliveira <
> profdouglaso.del...@gmail.com> escreveu:
>
>> Olá Pedro, primeiramente muito obrigado pela sua solução, eu dei uma
>> olhada rápida e acredito estar correta. Estarei olhando com mais calma,
>> assim que tiver um tempinho.
>>
>> Douglas Oliveira.
>>
>> Em dom, 8 de mar de 2020 11:05, Pedro José 
>> escreveu:
>>
>>> Bom dia!
>>> Não compreendi o porquê dessa questão ter sido vilipendiada. Não sou
>>> matemático, sou pitaqueiro, ouço falar em inteiros de Gauss vou atrás, de
>>> espaço fibrado idem, equações de Pell idem..., o que não consigo aprender
>>> fica para o futuro. Quando me aposentar  cursar uma faculdade de
>>> matemática. Portanto, nem tudo que resolvo me dá segurança. Reforço, alguém
>>> poderia me informar se está correto?
>>> Saudações,
>>> PJMS.
>>>
>>> Em ter, 3 de mar de 2020 12:03, Pedro José 
>>> escreveu:
>>>
>>>> Boa tarde!
>>>> Não me senti muito seguro na resposta. Está correto?
>>>>
>>>> Saudações,
>>>> PJMS
>>>>
>>>> Em seg., 2 de mar. de 2020 às 23:27, Pedro José 
>>>> escreveu:
>>>>
>>>>> Boa noite!
>>>>> Creio ter conseguido.
>>>>> Seja k o número de algarismos do período de 1/3^2005. Como (3,10)=1
>>>>> então k é a ordem 10 mod 3^2005.
>>>>> 3^(n-2)|| 3^(n-2); (|| significa divide exatamente) e 3^2||10-1 então
>>>>> pelo lema de Hensel 3^n||10^(3^(n-2))-1 para n>=2.(i)
>>>>> Então 10^(3^(n-2))= 1 mod 3^n logo ord 10 mod 3^n | 3^(n-2) Se
>>>>> x<>3^(n-2) absurdo; pois, teria que ser 3^k com k>>>> e por (i) 3^(k+2)||10^(3^k)-1 e k+2>>>> ord 10 mod 3^2005 =3^2003
>>>>> 3^2003 algarismos
>>>>> Saudações,
>>>>> PJMS
>>>>>
>>>>> Em sáb, 29 de fev de 2020 16:13, Pedro José 
>>>>> escreveu:
>>>>>
>>>>>> Boa tarde!
>>>>>> 3^2005 e não 10^2005.
>>>>>>
>>>>>> Em sex, 28 de fev de 2020 16:06, Pedro José 
>>>>>> escreveu:
>>>>>>
>>>>>>> Boa tarde!
>>>>>>> Questão complicada.
>>>>>>> Como (3^2005; 10) =1, o número de dígitos x deve ser a ordem de 10
>>>>>>> mod 10^2005. Portanto x | 2*3^2004.
>>>>>>> Se 10 fosse uma raiz primitiva de 3^2005 aí daria x=2.3^2004. Mas
>>>>>>> parece que não...
>>>>>>> Achar essa ordem é muito difícil, pelo menos para mim.
>>>>>>> O que achei empiricamente foi a conjectura: ord 10 mod 3^n = 3^(n-2)
>>>>>>> para n>=2.
>>>>>>> Será que sai por indução, aí seriam 3^2003 algarismos. Caso a
>>>>>>> conjectura esteja correta.
>>>>>>>
>>>>>>> Saudações,
>>>>>>> PJMS
>>>>>>>
>>>>>>> Em qui., 20 de fev. de 2020 às 18:12, Prof. Douglas Oliveira <
>>>>>>> profdouglaso.del...@gmail.com> escreveu:
>>>>>>>
>>>>>>>> Qual o número de dígitos do período de 1/(3^2005) ?
>>>>>>>>
>>>>>>>>
>>>>>>>> Saudações
>>>>>>>> Douglas Oliveira
>>>>>>>>
>>>>>>>> --
>>>>>>>> Esta mensagem foi verificada pelo sistema de antivírus e
>>>>>>>> acredita-se estar livre de perigo.
>>>>>>>
>>>>>>>
>>> --
>>> Esta mensagem foi verificada pelo sistema de antivírus e
>>> acredita-se estar livre de perigo.
>>
>>
> --
> Esta mensagem foi verificada pelo sistema de antivírus e
> acredita-se estar livre de perigo.

-- 
Esta mensagem foi verificada pelo sistema de antiv�rus e
 acredita-se estar livre de perigo.



[obm-l] Re: [obm-l] Ajuda com dízima

2020-03-08 Por tôpico Pedro José
Bom dia!
Não compreendi o porquê dessa questão ter sido vilipendiada. Não sou
matemático, sou pitaqueiro, ouço falar em inteiros de Gauss vou atrás, de
espaço fibrado idem, equações de Pell idem..., o que não consigo aprender
fica para o futuro. Quando me aposentar  cursar uma faculdade de
matemática. Portanto, nem tudo que resolvo me dá segurança. Reforço, alguém
poderia me informar se está correto?
Saudações,
PJMS.

Em ter, 3 de mar de 2020 12:03, Pedro José  escreveu:

> Boa tarde!
> Não me senti muito seguro na resposta. Está correto?
>
> Saudações,
> PJMS
>
> Em seg., 2 de mar. de 2020 às 23:27, Pedro José 
> escreveu:
>
>> Boa noite!
>> Creio ter conseguido.
>> Seja k o número de algarismos do período de 1/3^2005. Como (3,10)=1 então
>> k é a ordem 10 mod 3^2005.
>> 3^(n-2)|| 3^(n-2); (|| significa divide exatamente) e 3^2||10-1 então
>> pelo lema de Hensel 3^n||10^(3^(n-2))-1 para n>=2.(i)
>> Então 10^(3^(n-2))= 1 mod 3^n logo ord 10 mod 3^n | 3^(n-2) Se x<>3^(n-2)
>> absurdo; pois, teria que ser 3^k com k> e por (i) 3^(k+2)||10^(3^k)-1 e k+2> ord 10 mod 3^2005 =3^2003
>> 3^2003 algarismos
>> Saudações,
>> PJMS
>>
>> Em sáb, 29 de fev de 2020 16:13, Pedro José 
>> escreveu:
>>
>>> Boa tarde!
>>> 3^2005 e não 10^2005.
>>>
>>> Em sex, 28 de fev de 2020 16:06, Pedro José 
>>> escreveu:
>>>
>>>> Boa tarde!
>>>> Questão complicada.
>>>> Como (3^2005; 10) =1, o número de dígitos x deve ser a ordem de 10 mod
>>>> 10^2005. Portanto x | 2*3^2004.
>>>> Se 10 fosse uma raiz primitiva de 3^2005 aí daria x=2.3^2004. Mas
>>>> parece que não...
>>>> Achar essa ordem é muito difícil, pelo menos para mim.
>>>> O que achei empiricamente foi a conjectura: ord 10 mod 3^n = 3^(n-2)
>>>> para n>=2.
>>>> Será que sai por indução, aí seriam 3^2003 algarismos. Caso a
>>>> conjectura esteja correta.
>>>>
>>>> Saudações,
>>>> PJMS
>>>>
>>>> Em qui., 20 de fev. de 2020 às 18:12, Prof. Douglas Oliveira <
>>>> profdouglaso.del...@gmail.com> escreveu:
>>>>
>>>>> Qual o número de dígitos do período de 1/(3^2005) ?
>>>>>
>>>>>
>>>>> Saudações
>>>>> Douglas Oliveira
>>>>>
>>>>> --
>>>>> Esta mensagem foi verificada pelo sistema de antivírus e
>>>>> acredita-se estar livre de perigo.
>>>>
>>>>

-- 
Esta mensagem foi verificada pelo sistema de antiv�rus e
 acredita-se estar livre de perigo.



[obm-l] Re: [obm-l] Ajuda com dízima

2020-03-03 Por tôpico Pedro José
Boa tarde!
Não me senti muito seguro na resposta. Está correto?

Saudações,
PJMS

Em seg., 2 de mar. de 2020 às 23:27, Pedro José 
escreveu:

> Boa noite!
> Creio ter conseguido.
> Seja k o número de algarismos do período de 1/3^2005. Como (3,10)=1 então
> k é a ordem 10 mod 3^2005.
> 3^(n-2)|| 3^(n-2); (|| significa divide exatamente) e 3^2||10-1 então pelo
> lema de Hensel 3^n||10^(3^(n-2))-1 para n>=2.(i)
> Então 10^(3^(n-2))= 1 mod 3^n logo ord 10 mod 3^n | 3^(n-2) Se x<>3^(n-2)
> absurdo; pois, teria que ser 3^k com k e por (i) 3^(k+2)||10^(3^k)-1 e k+2 ord 10 mod 3^2005 =3^2003
> 3^2003 algarismos
> Saudações,
> PJMS
>
> Em sáb, 29 de fev de 2020 16:13, Pedro José 
> escreveu:
>
>> Boa tarde!
>> 3^2005 e não 10^2005.
>>
>> Em sex, 28 de fev de 2020 16:06, Pedro José 
>> escreveu:
>>
>>> Boa tarde!
>>> Questão complicada.
>>> Como (3^2005; 10) =1, o número de dígitos x deve ser a ordem de 10 mod
>>> 10^2005. Portanto x | 2*3^2004.
>>> Se 10 fosse uma raiz primitiva de 3^2005 aí daria x=2.3^2004. Mas parece
>>> que não...
>>> Achar essa ordem é muito difícil, pelo menos para mim.
>>> O que achei empiricamente foi a conjectura: ord 10 mod 3^n = 3^(n-2)
>>> para n>=2.
>>> Será que sai por indução, aí seriam 3^2003 algarismos. Caso a conjectura
>>> esteja correta.
>>>
>>> Saudações,
>>> PJMS
>>>
>>> Em qui., 20 de fev. de 2020 às 18:12, Prof. Douglas Oliveira <
>>> profdouglaso.del...@gmail.com> escreveu:
>>>
>>>> Qual o número de dígitos do período de 1/(3^2005) ?
>>>>
>>>>
>>>> Saudações
>>>> Douglas Oliveira
>>>>
>>>> --
>>>> Esta mensagem foi verificada pelo sistema de antivírus e
>>>> acredita-se estar livre de perigo.
>>>
>>>

-- 
Esta mensagem foi verificada pelo sistema de antiv�rus e
 acredita-se estar livre de perigo.



[obm-l] Re: [obm-l] Ajuda com dízima

2020-03-02 Por tôpico Pedro José
Boa noite!
Creio ter conseguido.
Seja k o número de algarismos do período de 1/3^2005. Como (3,10)=1 então k
é a ordem 10 mod 3^2005.
3^(n-2)|| 3^(n-2); (|| significa divide exatamente) e 3^2||10-1 então pelo
lema de Hensel 3^n||10^(3^(n-2))-1 para n>=2.(i)
Então 10^(3^(n-2))= 1 mod 3^n logo ord 10 mod 3^n | 3^(n-2) Se x<>3^(n-2)
absurdo; pois, teria que ser 3^k com k escreveu:

> Boa tarde!
> 3^2005 e não 10^2005.
>
> Em sex, 28 de fev de 2020 16:06, Pedro José 
> escreveu:
>
>> Boa tarde!
>> Questão complicada.
>> Como (3^2005; 10) =1, o número de dígitos x deve ser a ordem de 10 mod
>> 10^2005. Portanto x | 2*3^2004.
>> Se 10 fosse uma raiz primitiva de 3^2005 aí daria x=2.3^2004. Mas parece
>> que não...
>> Achar essa ordem é muito difícil, pelo menos para mim.
>> O que achei empiricamente foi a conjectura: ord 10 mod 3^n = 3^(n-2) para
>> n>=2.
>> Será que sai por indução, aí seriam 3^2003 algarismos. Caso a conjectura
>> esteja correta.
>>
>> Saudações,
>> PJMS
>>
>> Em qui., 20 de fev. de 2020 às 18:12, Prof. Douglas Oliveira <
>> profdouglaso.del...@gmail.com> escreveu:
>>
>>> Qual o número de dígitos do período de 1/(3^2005) ?
>>>
>>>
>>> Saudações
>>> Douglas Oliveira
>>>
>>> --
>>> Esta mensagem foi verificada pelo sistema de antivírus e
>>> acredita-se estar livre de perigo.
>>
>>

-- 
Esta mensagem foi verificada pelo sistema de antiv�rus e
 acredita-se estar livre de perigo.



[obm-l] Re: [obm-l] Ajuda com dízima

2020-02-29 Por tôpico Pedro José
Boa tarde!
3^2005 e não 10^2005.

Em sex, 28 de fev de 2020 16:06, Pedro José  escreveu:

> Boa tarde!
> Questão complicada.
> Como (3^2005; 10) =1, o número de dígitos x deve ser a ordem de 10 mod
> 10^2005. Portanto x | 2*3^2004.
> Se 10 fosse uma raiz primitiva de 3^2005 aí daria x=2.3^2004. Mas parece
> que não...
> Achar essa ordem é muito difícil, pelo menos para mim.
> O que achei empiricamente foi a conjectura: ord 10 mod 3^n = 3^(n-2) para
> n>=2.
> Será que sai por indução, aí seriam 3^2003 algarismos. Caso a conjectura
> esteja correta.
>
> Saudações,
> PJMS
>
> Em qui., 20 de fev. de 2020 às 18:12, Prof. Douglas Oliveira <
> profdouglaso.del...@gmail.com> escreveu:
>
>> Qual o número de dígitos do período de 1/(3^2005) ?
>>
>>
>> Saudações
>> Douglas Oliveira
>>
>> --
>> Esta mensagem foi verificada pelo sistema de antivírus e
>> acredita-se estar livre de perigo.
>
>

-- 
Esta mensagem foi verificada pelo sistema de antiv�rus e
 acredita-se estar livre de perigo.



[obm-l] Re: [obm-l] Ajuda com dízima

2020-02-28 Por tôpico Pedro José
Boa tarde!
Questão complicada.
Como (3^2005; 10) =1, o número de dígitos x deve ser a ordem de 10 mod
10^2005. Portanto x | 2*3^2004.
Se 10 fosse uma raiz primitiva de 3^2005 aí daria x=2.3^2004. Mas parece
que não...
Achar essa ordem é muito difícil, pelo menos para mim.
O que achei empiricamente foi a conjectura: ord 10 mod 3^n = 3^(n-2) para
n>=2.
Será que sai por indução, aí seriam 3^2003 algarismos. Caso a conjectura
esteja correta.

Saudações,
PJMS

Em qui., 20 de fev. de 2020 às 18:12, Prof. Douglas Oliveira <
profdouglaso.del...@gmail.com> escreveu:

> Qual o número de dígitos do período de 1/(3^2005) ?
>
>
> Saudações
> Douglas Oliveira
>
> --
> Esta mensagem foi verificada pelo sistema de antivírus e
> acredita-se estar livre de perigo.

-- 
Esta mensagem foi verificada pelo sistema de antiv�rus e
 acredita-se estar livre de perigo.



[obm-l] Re: [obm-l] Ângulos em um triângulo

2020-02-13 Por tôpico Pedro Cardoso
Deve haver um jeito mais fácil, mas foi o que eu pensei agora

Construa os circumcírculos de ABM e NBC. Pela lei dos senos, eles têm o
mesmo raio.
Seja X o centro do circuncírculo de ABM, e Y o de NBC.
B está na intersersão dos circumcírculos, então B está na mediatriz de XY.
AXM, NYC e XBY são isósceles.
ABC e MBN são isósceles
O pé da altura de B em relação a MN coincide com Q.
BQC=90°

Em qui, 13 de fev de 2020 22:42, Vanderlei Nemitz 
escreveu:

> Boa noite!
>
> Usei várias leis dos senos, obtive coisas legais, mas não o ângulo pedido.
> Alguém conhece algo interessante?
>
>
>
> Muito obrigado!
>
>
>
> *Em um triângulo ABC, os pontos consecutivos M, Q, N do lado AC são tais
> que AM = NC. Se Q é ponto médio de MN e os ângulos NBC e ABM medem 20º,
> calcule a medida do ângulo BQC.*
>
> --
> Esta mensagem foi verificada pelo sistema de antivírus e
> acredita-se estar livre de perigo.

-- 
Esta mensagem foi verificada pelo sistema de antiv�rus e
 acredita-se estar livre de perigo.



[obm-l] Re: [obm-l] Re: [obm-l] Cálculo do Volume de um Sólido

2020-02-12 Por tôpico Pedro José
Bom dia!
Alguém poderia me ajudar e mostrar onde errei os limites? Resolvendo por
integral tripla, usando f(x,y,z)=1.

Grato,
PJMS

Em ter, 11 de fev de 2020 13:11, Pedro José  escreveu:

> Boa tarde!
>
> Resolvi por método numérico usando, pelo menos penso eu, os mesmos limites
> e encontrei 2,1329, muito próximo da resposta. Gostaria que alguém me
> ajudasse onde errei na integral tripla.
> Usei z^2-y e 2z-y como os limites para integral em dx. Em seguida, z^2 e
> 2z para dy e finalmente 0 e 2 para dz.
> Onde está o erro?
> Grato,
> PJMS
>
> Em ter, 11 de fev de 2020 12:49, Claudio Buffara <
> claudio.buff...@gmail.com> escreveu:
>
>> O sólido é a região do 1o octante (todas as coordenadas positivas)
>> compreendida entre os planos x-z e y-z, acima do plano z = (x+y)/2 e abaixo
>> da z = raiz(x+y).
>> A superfície e o plano se intersectam numa reta:
>> raiz(x+y) = (x+y)/2 ==> x+y = (x+y)^2/4 ==> x+y = 4, contida no plano z =
>> 2.
>>
>> Assim, o volume pode ser dado pela diferença entre duas integrais duplas,
>> calculadas sobre o domínio D, no plano x-y, dado por x > 0, y > 0 e x+y = 4.
>> Volume = Integral(D) raiz(x+y)*dA - Integral(D) (x+y)/2*dA.
>>
>> Usando coordenadas cartesianas, a primeira integral fica:
>> Integral(x=0...4)Integral(y=0...4-x)*raiz(x+y)*dy*dx
>> = Integral(0...4) (2/3)*(4^(3/2) - x^(3/2))*dx
>> = Integral(0...4) (16/3 - (2/3)*x^(3/2))
>> = 64/3 - (4/15)*4^(5/2)
>> = 64/3 - 128/15
>> = 64/5
>>
>> A segunda integral é:
>> Integral(x=0...4)Integral(y=0...4-x) (x+y)/2*dy*dx
>> = Integral(x=0...4) (1/2)*(x*(4-x) + (4-x)^2/2)*dx
>> = Integral(0...4) (4 - x^2/4)*dx
>> = 32/3
>>
>> Logo, o volume é 64/5 - 32/3 = 32/15  (se não errei nenhuma conta...)
>>
>> []s,
>> Claudio.
>>
>>
>> On Mon, Feb 3, 2020 at 8:55 PM Luiz Antonio Rodrigues <
>> rodrigue...@gmail.com> wrote:
>>
>>> Olá, pessoal!
>>> Tudo bem?
>>> Estou tentando resolver o seguinte problema:
>>>
>>> Ache o volume da região tridimensional definida por:
>>>
>>> z^2>>
>>> Sendo que:
>>> x>0 e y>0 e z>0
>>>
>>> Com o auxílio de um software eu consegui visualizar o sólido em questão.
>>> Eu calculei o volume do sólido girando em torno do eixo z e dividindo o
>>> resultado por 4.
>>> A resposta que eu obtive foi (16*pi)/15, que não está correta.
>>> Já refiz os cálculos muitas vezes e chego sempre na mesma resposta.
>>> Alguém pode me ajudar?
>>> Muito obrigado e um abraço!
>>>
>>> --
>>> Esta mensagem foi verificada pelo sistema de antivírus e
>>> acredita-se estar livre de perigo.
>>
>>
>> --
>> Esta mensagem foi verificada pelo sistema de antivírus e
>> acredita-se estar livre de perigo.
>
>

-- 
Esta mensagem foi verificada pelo sistema de antiv�rus e
 acredita-se estar livre de perigo.



[obm-l] Re: [obm-l] Re: [obm-l] Cálculo do Volume de um Sólido

2020-02-11 Por tôpico Pedro José
Boa tarde!

Resolvi por método numérico usando, pelo menos penso eu, os mesmos limites
e encontrei 2,1329, muito próximo da resposta. Gostaria que alguém me
ajudasse onde errei na integral tripla.
Usei z^2-y e 2z-y como os limites para integral em dx. Em seguida, z^2 e 2z
para dy e finalmente 0 e 2 para dz.
Onde está o erro?
Grato,
PJMS

Em ter, 11 de fev de 2020 12:49, Claudio Buffara 
escreveu:

> O sólido é a região do 1o octante (todas as coordenadas positivas)
> compreendida entre os planos x-z e y-z, acima do plano z = (x+y)/2 e abaixo
> da z = raiz(x+y).
> A superfície e o plano se intersectam numa reta:
> raiz(x+y) = (x+y)/2 ==> x+y = (x+y)^2/4 ==> x+y = 4, contida no plano z =
> 2.
>
> Assim, o volume pode ser dado pela diferença entre duas integrais duplas,
> calculadas sobre o domínio D, no plano x-y, dado por x > 0, y > 0 e x+y = 4.
> Volume = Integral(D) raiz(x+y)*dA - Integral(D) (x+y)/2*dA.
>
> Usando coordenadas cartesianas, a primeira integral fica:
> Integral(x=0...4)Integral(y=0...4-x)*raiz(x+y)*dy*dx
> = Integral(0...4) (2/3)*(4^(3/2) - x^(3/2))*dx
> = Integral(0...4) (16/3 - (2/3)*x^(3/2))
> = 64/3 - (4/15)*4^(5/2)
> = 64/3 - 128/15
> = 64/5
>
> A segunda integral é:
> Integral(x=0...4)Integral(y=0...4-x) (x+y)/2*dy*dx
> = Integral(x=0...4) (1/2)*(x*(4-x) + (4-x)^2/2)*dx
> = Integral(0...4) (4 - x^2/4)*dx
> = 32/3
>
> Logo, o volume é 64/5 - 32/3 = 32/15  (se não errei nenhuma conta...)
>
> []s,
> Claudio.
>
>
> On Mon, Feb 3, 2020 at 8:55 PM Luiz Antonio Rodrigues <
> rodrigue...@gmail.com> wrote:
>
>> Olá, pessoal!
>> Tudo bem?
>> Estou tentando resolver o seguinte problema:
>>
>> Ache o volume da região tridimensional definida por:
>>
>> z^2>
>> Sendo que:
>> x>0 e y>0 e z>0
>>
>> Com o auxílio de um software eu consegui visualizar o sólido em questão.
>> Eu calculei o volume do sólido girando em torno do eixo z e dividindo o
>> resultado por 4.
>> A resposta que eu obtive foi (16*pi)/15, que não está correta.
>> Já refiz os cálculos muitas vezes e chego sempre na mesma resposta.
>> Alguém pode me ajudar?
>> Muito obrigado e um abraço!
>>
>> --
>> Esta mensagem foi verificada pelo sistema de antivírus e
>> acredita-se estar livre de perigo.
>
>
> --
> Esta mensagem foi verificada pelo sistema de antivírus e
> acredita-se estar livre de perigo.

-- 
Esta mensagem foi verificada pelo sistema de antiv�rus e
 acredita-se estar livre de perigo.



[obm-l] Re: [obm-l] Re: [obm-l] Re: [obm-l] Re: [obm-l] Análise complexa - mostrar que f é sobrejetora

2020-02-10 Por tôpico Pedro Angelo
Aparentemente, errei hehe. Achei engraçada essa explicação: funções
holomorfas não-inteiras também têm "série de potências inteiras" em
torno de cada ponto. Por que só as inteiras levam o nome?

Le lun. 10 févr. 2020 à 20:52, Bernardo Freitas Paulo da Costa
 a écrit :
>
> On Mon, Feb 10, 2020 at 8:16 PM Artur Costa Steiner
>  wrote:
> > O adjetivo inteira, em análise complexa,  não tem nada a ver com o que ele 
> > sugere. Acho uma terminologia infeliz, mas é consagrada.
>
> Um chute: em francês, o termo "série inteira" (por oposição a série
> fracionária) se refere às séries de potências (inteiras) da variável z
> (por oposição às "séries de Puiseux" onde há expoentes fracionários).
> E as funções inteiras têm expansão, convergente, como série de
> potências (inteiras) da variável z, f(z) = \sum_{n=0}^\infty a_n z^n.
>
> Abraços,
> --
> Bernardo Freitas Paulo da Costa
>
> --
> Esta mensagem foi verificada pelo sistema de antivírus e
>  acredita-se estar livre de perigo.
>
>
> =
> Instru�ões para entrar na lista, sair da lista e usar a lista em
> http://www.mat.puc-rio.br/~obmlistas/obm-l.html
> =

-- 
Esta mensagem foi verificada pelo sistema de antiv�rus e
 acredita-se estar livre de perigo.


=
Instru��es para entrar na lista, sair da lista e usar a lista em
http://www.mat.puc-rio.br/~obmlistas/obm-l.html
=


[obm-l] Re: [obm-l] Re: [obm-l] Re: [obm-l] Análise complexa - mostrar que f é sobrejetora

2020-02-10 Por tôpico Pedro Angelo
Eu gosto de pensar o "inteira" como significando que a série de
potências f(z) = a_0 + a_1 z + ... converge no plano *inteiro*.

Le lun. 10 févr. 2020 à 20:16, Artur Costa Steiner
 a écrit :
>
>
>
> Em seg, 10 de fev de 2020 17:28, Anderson Torres 
>  escreveu:
>>
>> Em dom., 9 de fev. de 2020 às 21:50, Artur Costa Steiner
>>  escreveu:
>> >
>> > Nunca vi este curioso fato ser citado em lugar nenhum.É fácil de provar 
>> > recorrendo-se ao teorema de Picard. Será que há uma prova simples (ou uma 
>> > qualquer) que não recorra a este teorema?
>> >
>> > Se a não identicamente nula f for inteira e ímpar, então f é sobrejetora.
>> >
>>
>> O que é função inteira?
>>
>> Se f é uma função definida em um aberto V do plano complexo C, dizemos que f 
>> é holomorfa em V se f for diferenciável em cada elemento de V.
>
>
> Se V = C, dizemos que f é inteira. Assim, uma função de C em C é inteira 
> se for diferenciável em todo o C. É holomorfa em C.
>
> O adjetivo inteira, em análise complexa,  não tem nada a ver com o que ele 
> sugere. Acho uma terminologia infeliz, mas é consagrada.
>
> Artur
>
>
>
>
> --
> Esta mensagem foi verificada pelo sistema de antivírus e
> acredita-se estar livre de perigo.

-- 
Esta mensagem foi verificada pelo sistema de antiv�rus e
 acredita-se estar livre de perigo.


=
Instru��es para entrar na lista, sair da lista e usar a lista em
http://www.mat.puc-rio.br/~obmlistas/obm-l.html
=


[obm-l] Re: [obm-l] Re: [obm-l] Re: [obm-l] Re: [obm-l] Cálculo do Volume de um Sólido

2020-02-10 Por tôpico Pedro José
Boa noite!
Não sei onde errei está dando exatamente a metade 16/15.
Saudações,
PJMS

Em seg, 10 de fev de 2020 15:46, Luiz Antonio Rodrigues <
rodrigue...@gmail.com> escreveu:

> Olá, Pedro!
> Tudo bem?
> Obrigado pela resposta!
> A resposta realmente não tem pi: é 32/15.
> Eu percebi ontem que o meu erro foi fazer uma rotação em torno do eixo z.
> Se seccionarmos a figura no plano xy teremos um trapézio.
> Vou pensar na sua sugestão e tentar fazer tudo de novo.
> Muito obrigado!
> Abraços!
> Luiz
>
>
>
> Em seg, 10 de fev de 2020 1:38 PM, Pedro José 
> escreveu:
>
>> Boa tarde!
>> Como no caso você tem a resposta, facilitaria se a expusesse.
>> Para evitar que postemos soluções erradas.
>>
>> Saudações,
>> PJMS
>>
>> Em qui., 6 de fev. de 2020 às 07:41, Anderson Torres <
>> torres.anderson...@gmail.com> escreveu:
>>
>>> Em seg., 3 de fev. de 2020 às 20:55, Luiz Antonio Rodrigues
>>>  escreveu:
>>> >
>>> > Olá, pessoal!
>>> > Tudo bem?
>>> > Estou tentando resolver o seguinte problema:
>>> >
>>> > Ache o volume da região tridimensional definida por:
>>> >
>>> > z^2>> >
>>> > Sendo que:
>>> > x>0 e y>0 e z>0
>>> >
>>> > Com o auxílio de um software eu consegui visualizar o sólido em
>>> questão.
>>> > Eu calculei o volume do sólido girando em torno do eixo z e dividindo
>>> o resultado por 4.
>>> > A resposta que eu obtive foi (16*pi)/15, que não está correta.
>>> > Já refiz os cálculos muitas vezes e chego sempre na mesma resposta.
>>> > Alguém pode me ajudar?
>>>
>>> Tem como cê enviar as contas e o desenho que cê fez?
>>>
>>> > Muito obrigado e um abraço!
>>> >
>>> > --
>>> > Esta mensagem foi verificada pelo sistema de antivírus e
>>> > acredita-se estar livre de perigo.
>>>
>>> --
>>> Esta mensagem foi verificada pelo sistema de antivírus e
>>>  acredita-se estar livre de perigo.
>>>
>>>
>>> =
>>> Instru�ões para entrar na lista, sair da lista e usar a lista em
>>> http://www.mat.puc-rio.br/~obmlistas/obm-l.html
>>> =
>>>
>>
>> --
>> Esta mensagem foi verificada pelo sistema de antivírus e
>> acredita-se estar livre de perigo.
>
>
> --
> Esta mensagem foi verificada pelo sistema de antivírus e
> acredita-se estar livre de perigo.

-- 
Esta mensagem foi verificada pelo sistema de antiv�rus e
 acredita-se estar livre de perigo.



  1   2   3   4   5   6   7   8   9   10   >